Está en la página 1de 30

Centro de Estudios Aula Neptuno - GRANADA Ver.

: 07/2000
www.aulaneptuno.com
- 1 -
1. La fractura del maleolo externo por aduccin del
pie se denomina :

1. Tillaux.
2. Dupuytren.
3. Maisonneuve.
4. Sever.
5. Wagstaffe.

2. Indica la incorrecta con respecto a la fractura
supracondlea de hmero en el nio:

1. Est asociada con el grave riesgo de una isquemia
de Volkmann.
2. El 90% de los casos son fracturas de tipo "en
flexin".
3. Puede producirse en hiperextensin o por cada
sobre la mano con el codo flexionado.
4. El fragmento distal suele estar por detrs.
5. El fragmento distal suele desplazarse ms a me-
nudo hacia dentro.

3. Las fracturas diafisiarias ms frecuentes son las
localizadas en :

1. Fmur.
2. Hmero.
3. Tibia.
4. Cbito.
5. Radio.

4. Las fracturas metafisarias ms frecuentes se loca-
lizan en :

1. Fmur.
2. Hmero.
3. Tibia.
4. Cbito.
5. Radio.

5. Ante un joven de sexo masculino con fractura
patolgica en metfisis superior del hmero hay
que pensar en:

1. Quiste seo solitario.
2. Tumor glmico.
3. Condroma.
4. Osteoma osteoide.
5. Tumor de Ewing.

6. La enfermedad de BLOUNT origina:

1. Tibia en valgo.
2. Coxa vara.
3. Genu varo.
4. Genu valgo.
5. Coxa valga.

7. Cul de los siguientes tumores malignos se carac-
teriza por su mayor radiosensibilidad? :

1. Fibrosarcoma.
2. Reticulosarcoma.
3. Condrosarcoma.
4. Sarcoma de Ewing.
5. Osteosarcoma.

8. Varn de 25 aos en el que se descubre una lesin
osteoblstica con calcificacin interna a nivel de la
metfisis inferior de la tibia. El nico dato de la
anamnesis reseable es la presencia de dolor inten-
so en esa localizacin que empeora con el reposo y
tiene un predominio nocturno. Cul sera tu dia-
gnstico de sospecha? :

1. Osteoma.
2. Osteocondroma.
3. Quiste seo solitario.
4. Osteoma Osteoide.
5. Exstosis cartilaginosa.

9. Mujer de 40 aos sin antecedentes personales de
inters que es remitida a un servicio de urgencias
trs ser rescatada de su domicilio al haberse pro-
ducido un incendio. Uno de los siguientes datos de
la exploracin fsica o de laboratorio NO encon-
trara correspondencia con su cuadro :

1. Somnolencia.
2. Carboxihemoglobina del 8 %.
3. Debilidad muscular.
4. Neuritis ptica secuelar.
5. PaO2 de 40 mm Hg.

10. Un enfermo ingresa en el Servicio de Urgencias
con nuseas, vmitos, dolor epigstrico, sudor, sali-
vacin, dolor subesternal y disnea. La exploara-
cin manifiesta bloqueo cardiaco de conduccin y
dificultad de acomodacin. Como antecedente fi-
gura una excursin campestre con ingestin de se-
tas una hora antes de aparecer la sintomatologa.
Instaurar Vd. un tratamiento con :

1. Paratin.
2. Metacolina.
3. Atropina por va iv.
4. Fisostigmina.
5. Dopamina.

11. En el estadio II del cancer de ovario, se seala
como correcto el siguiente tratamiento:

1. Cirugia conservadora y radioterapia
2. Quimioterapia
3. Radioterapia
4. Cirugia radical mas quimioterapia
5. Biopsia del tumor y quimioterapia

12. Ante una paciente con amenorrea primaria que
presenta ausencia de vagina y cariotipo 46 XY el
diagnostico sera:

1. Sindrome de Kallman
2. Sindrome de Asherman
3. Sindrome de Morris
4. Sindrome de Rokitansky
5. Ninguno de los anteriores

Centro de Estudios Aula Neptuno - GRANADA Ver.: 07/2000
www.aulaneptuno.com
- 2 -
13. El Sindrome de ovario poliquistico se caracteriza
por todo lo siguiente EXCEPTO por:

1. Aumento de la LH
2. Aumento de la estrona
3. Hiperplasia de la teca
4. Anovulacion
5. Disminucion de la testosterona

14. La enfermedad inflamatorio pelvica se caracteriza
clnicamente por los siguientes datos excepto:

1. Dolor espontneo a la palpacin del abdomen in-
ferior.
2. Signos de irritacin peritoneal.
3. Dolor a la movilizacin vertical.
4. Dolor anexial.
5. Hemorragia vaginal.

15. Las infecciones tempranas de la herida quirrgica
(24-48 horas tras la ciruga) suelen estar causadas
por:

1. Pseudomonas.
2. St.epidermidis.
3. St.aureus.
4. Str.pyogenes.
5. Candida.

16. Una de estas caractersticas NO es propia de la
Escarlatina :

1. Lengua aframbuesada.
2. Descamacin de la piel.
3. Eritema palmoplantar.
4. Enantema.
5. Tumefaccin folicular.

17. Paciente intervenido por estenosis artica, median-
te sustitucin valvular, que en el postoperatorio
presenta fiebre elevada, hipotensin y taquicardia.
En el hemocultivo se asla un germen. Lo ms
probable es que ste:

1. Produzca dextrano.
2. Sea coagulasa (-).
3. Sea Gram (-).
4. Tenga forma espirilar.
5. Crezca en medios con bilis.

18. En el tratamiento antibitico del anterior paciente
emplearemos como primera opcin:

1. Gentamicina.
2. Ampicilina.
3. Penicilina G.
4. Vancomicina.
5. Carbenicilina.

19. Una de estas medidas no debe emplearse en una
enteritis por shigella.

1. Loperamida.
2. Hidratacin.
3. Reposo digestivo.
4. Acido nalidxico.
5. Norfloxacino.

20. Un nio de 4 aos es llevado a urgencias por un
cuadro agudo de fiebre y odinofagia, que en pocas
horas se ha complicado con disfagia, sialorrea y
disnea con estridor. La intensidad de la voz del ni-
o ha disminuido, pero no hay disfona. La inspec-
cin de la faringe es normal. El agente infeccioso
ms probablemente responsable del cuadro es:

1. Virus respiratorio sincitial.
2. Virus parainfluenzae.
3. Mycoplasma pneumoniae.
4. Haemophilus influenzae.
5. Neumococo.

21. Paciente de 72 aos, en tratamiento con corticoides
por enfermedad ampollosa, que presenta cuadro
prolongado con febrcula, astenia, anorexia y pr-
dida de peso. A la exploracin slo destaca una
hepatoesplenomegalia, y en la analtica: Ht: 2,9 x
1012 Ht/L, Leu: 43.000/mL, plaq: 71.000/mL,
VSG: 131-168, Fosfatasa alcalina: 769 U/L. La se-
rologa a bacterias y virus es negativa, no se consi-
gue aislar ningn germen de sangre, LCR, esputo
ni orina. El PPD es negativo. En la biopsia hepti-
ca se hallan numerosos focos necrticos con abun-
dantes bacilos acido-alcohol resistentes. El
dgto.ms probable es:

1. Tuberculosis miliar.
2. Nocardiosis.
3. Infeccin por Mycobacterium avium intracellulla-
re.
4. Tuberculosis crptica.
5. Lepra lepromatosa.

22. Paciente de 17 aos que presenta fiebre elevada de
comienzo brusco con cefaleas, vmitos y dolores
lumbares, seguida 2 das despus de exantema pe-
tequial palmo-plantar, artralgias importantes. Des-
taca como antecedente epidemiolgico de inters la
ingestin de leche cruda tres das antes del inicio de
la fiebre. La primera sospecha diagnstica debe
ser:

1. Botulismo.
2. Enfermedad de LYME.
3. Infeccin por estreptobacillus moliniformis.
4. Leptospirosis.
5. Brucelosis.

23. Cul de las siguientes rickettsiosis es ms frecuen-
te en Espaa?

1. Fiebre Q.
2. Tifus murino.
3. Fiebre de las trincheras.
4. Tifus de las malezas.
5. Fiebre botonosa.

24. Una de las pautas ms comunmente aceptadas en
Centro de Estudios Aula Neptuno - GRANADA Ver.: 07/2000
www.aulaneptuno.com
- 3 -
la actualidad para el tratamiento de las infecciones
diseminadas por M.avium intracellulare en los pa-
cientes con SIDA incluye cuatro de los siguientes
frmacos. Seala el error:

1. Ciprofloxacino.
2. Etambutol.
3. Rifampicina.
4. Teicoplanina.
5. Claritromicina.

25. Vanessa es en la actualidad una mujer de 28 aos.
Consult por primera vez cuando tena 25 porque
se estaba quedando sorda. Dos meses antes fue vis-
ta por su mdico de cabecera por una parlisis fa-
cial perifrica. La trat con corticosteroides duran-
te unos das recobrando su fisonoma normal con
rapidez. Dos semanas antes de la consulta al servi-
cio de ORL comenz a notar una prdida de la
audicin que se hizo progresiva. La audiometra
demostr una sordera perceptiva bilateral mucho
ms intensa en el odo izquierdo. Tena anteceden-
tes de adiccin a la herona y viva de la prostitu-
cin en una zona agrcola de Burgos. La explora-
cin fsica general fue normal. Los anticuerpos
frente a VIH resultaron positivos, tanto los anti-
env como los anti-gag. El Ag.VIH fue negativo y no
tena anormalidades inmunolgicas (CD4 de
850/mm3). Cul es el dgto.ms probable?

1. Meningitis criptoccica.
2. Infeccin primaria por VIH.
3. Complejo demencia-SIDA.
4. Sfilis.
5. Infeccin por CMV del SNC.

26. Qu afirmacin es cierta en las distintas manifes-
taciones de la tuberculosis postprimarias?

1. La respuesta de la adenitis tuberculosa al trata-
miento antituberculoso es prcticamente universal.
2. La tuberculosis intestinal se localiza de forma pre-
dominante en la unin rectosigmoidea.
3. En la mayora de los casos de tuberculosis osteoar-
ticular la infeccin se ha producido por contigi-
dad a partir del foco pulmonar.
4. La tuberculosis genitourinaria es la forma ms
precoz de presentacin y suele aparecer a los 6-7
meses de la primoinfeccin.
5. El tratamiento de la meningitis tuberculosa debe
instaurarse lo ms rpidamente posible, por lo que
no se debe esperar al diagnstico de certeza.

27. Los adenovirus no suelen provocar:

1. Cistitis hemorrgica.
2. Gastroenteritis aguda.
3. Queratoconjuntivitis epidmica.
4. Infeccin en trasplantados renales.
5. Endocarditis sobre vlvula protsica.

28. Cul es la conducta ms adecuada que se debe
adoptar ante un paciente VIH positivo que ha des-
pertado la sospecha clnica de presentar una toxo-
plasmosis del Sistema Nervioso Central?

1. Debe iniciarse tratamiento con pirimetamina y sul-
fadiacina; si no hay respuesta al mismo ser con-
veniente la realizacin de una TC o una RM y s-
lo si las mismas no son concluyentes se practicar
la biopsia cerebral.
2. Debemos intentar confirmar el diagnstico me-
diante TC o RM con lo cual, de ser positivas, ten-
dremos suficientes datos para iniciar el tratamien-
to; slo en pacientes que no respondan al mismo 2
a 4 semanas despus ser necesaria la realizacin
de biopsia cerebral.
3. Dados los importantes efectos secundarios del tra-
tamiento es necesario confirmar con absoluta cer-
teza el diagnstico con RM, TC y biopsia antes de
pautar la pirimetamina-sulfadiacina.
4. Se confirmar de entrada el diagnstico con RM
TC y se iniciar tratamiento con pirimetamina sul-
fadiacina, indicndose la biopsia cerebral con ms
tranquilidad para asegurarnos completamente.
5. Realizaremos de entrada estudio serolgico que
nos permitir confirmar con certeza el diagnstico;
seguidamente instauraremos tratamiento que slo
si no da resultados nos llevar a indicar la TC o la
RM.

29. La complicacin ms frecuente de la varicela es:

1. Otitis media.
2. Neumona varicelosa.
3. Encefalitis cerebelosa.
4. Artritis.
5. Infeccin cutnea secundaria.

30. Una mujer de 23 aos, en su primer trimestre de
gestacin, asiste a una comida en las afueras de la
ciudad, y durante la misma coge y consuela a un
nio llorn; posteriormente se sabe que ese nio
tena la rubeola. Se realiza una prueba de inhibi-
cin de la hemaglutinacin con la sangre de la mu-
jer, que da resultado negativo. En estas circunstan-
cias se debe:

1. Proponer la posibilidad de un aborto, si la madre
est de acuerdo.
2. Administrar arabinsido de citosina.
3. Administrar la vacuna de la rubeola y gammaglo-
bulina.
4. Administrar inmunoglobulina srica humana.
5. Esperar dos o tres semanas y repetir la prueba.

31. Cul de estos parsitos produce cor pulmonale?

1. Clonorquis.
2. Esquistosoma.
3. Filarias.
4. Necator.
5. Oncocerca.

32. Nio de Venezuela con fiebre, adenopatas, esple-
nomegalia, conjuntivitis, edema palpebral y ade-
nopata satlite. Sospecha:

Centro de Estudios Aula Neptuno - GRANADA Ver.: 07/2000
www.aulaneptuno.com
- 4 -
1. Amebiasis.
2. Paludismo.
3. Loasis.
4. Bartonelosis.
5. Tripanosomiasis.

33. Un varn de 45 aos, homosexual, consult por
hemiparesia progresiva. Es VIH (+) conocido des-
de seis meses antes. Haba notado una prdida de
peso pero haca una vida normal hasta una sema-
na antes del ingreso en que empez a notar aton-
tamiento, cefalea y prdida de destreza en la mano
izquierda. La noche antes de la consulta tuvo una
convulsin generalizada. Exploracin: El fondo de
ojo demostr signos incipientes de hipertensin
endocraneal. Tena hemiparesia izquierda con re-
flejos normales. La TAC craneal mostr varias le-
siones hipodensas rodeadas de anillo hipercaptante
y edema perilesional. Cul es la actitud ms con-
veniente con este paciente?

1. Biopsia cerebral inmediata.
2. Estudios serolgicos de toxoplasma antes de deci-
dir el tratamiento.
3. Puncin lumbar para cultivo, y serologa a cripto-
coco.
4. Tratamiento emprico con pirimetamina y sulfa-
diacina.
5. Tratamiento con dexametasona.

34. Despus de un viaje de placer a varios pases de
Africa, un intrpido joven de Guadalajara (Espa-
a), presenta un cuadro clnico que ha evoluciona-
do como un sndrome febril con malestar general,
con dolor intenso diferido a la palpacin sobre su-
perficies seas, seguido de linfadenopatas en la re-
gin cervical posterior y supraclavicular, y final-
mente somnolencia, signos extrapiramidales y
ataxia. El tratamiento ms eficaz en el paciente
descrito es:

1. Nitrofurazona.
2. Suramina.
3. Dietilcarbamacina.
4. Glucantime.
5. Melarsoprol.

35. Ante la sospecha diagnstica clnico-radiolgica de
neumona por Pn.carinii, la primera prueba que se
debe realizar para confirmar el diagnstico debe
ser:

1. Biopsia transbronquial.
2. Reaccin de fijacin del complemento.
3. Fibrobroncoscopia y lavado broncoalveolar.
4. Biopsia pulmonar abierta.
5. Estudio del esputo inducido.

36. Cul es el tratamiento ms recomendable ante
una herida contaminada por clostridios?

1. No necesita en principio tratamiento antibitico.
2. Bencilpenicilina.
3. Cloramfenicol.
4. Cefoxitina.
5. Metronidazol.

37. Entre las siguientes anormalidades inmunolgicas,
cul no es debida a infeccin por el VIH?

1. Disminucin de mediadores de la inmunorregula-
cin.
2. Descenso del cociente linfocitario CD4/CD8.
3. Disminucin de la proliferacin linfocitaria al es-
tmulo con lecitinas.
4. Disminucin de las inmunoglobulinas sricas.
5. Descenso del nmero absoluto de linfocitos CD4.

38. En relacin a la Epidemiologa de la infeccin por
VIH seala la respuesta falsa:

1. En Espaa se ha registrado, en los ltimos dos
aos, un importante incremento del nmero de ca-
sos de SIDA y de la mortalidad en los mismos.
2. La va ms frecuente de transmisin a nivel mun-
dial es la heterosexual.
3. En Espaa la va de transmisin ms frecuente si-
gue siendo la drogadiccin intravenosa con
predominio masculino, ya que el 80% de los casos
actuales son varones.
4. La infeccin oportunista ms frecuente en el SIDA
es, en la actualidad, la tuberculosis.
5. Ms del 90% de los infectados actualmente por el
VIH pertenecen a pases subdesarrollados.

39. Se han encontrado diversos serotipos de VIH-1,
demostrndose una diferencia geogrfica en su
distribucin y algunas caractersticas propias de
cada uno en cuanto a transmisin. Por ejemplo,
parece que predomina en Asia el serotipo que ma-
yor facilidad tiene para ser transmitido por va
heterosexual y que se denomina por la letra:

1. e.
2. d.
3. c.
4. b.
5. a.

40. Cul de los siguientes grmenes est ms frecuen-
temente implicado en la embolia sptica pulmonar
mltiple?

1. Estafilococo aureus.
2. Enterococo.
3. Neumococo.
4. Pneumocystis carinii.
5. Citomegalovirus.

41. Todas menos una se consideran signos de maligni-
zacin de la retinopata hipertensiva:

1. Exudados algodonosos.
2. Hemorragias retinianas.
3. Edema retiniano.
4. Depsitos de pigemnto en el rea macular.
5. Edema de papila.

Centro de Estudios Aula Neptuno - GRANADA Ver.: 07/2000
www.aulaneptuno.com
- 5 -
42. Paciente con enrojecimiento ocular,dolor, fotofobia
y disminucion de agudeza visual. Cual de las si-
guientes no es diagnostico diferencial:

1. Queratitis herpetica.
2. Glaucoma agudo de angulo cerrado.
3. Iridociclitis aguda.
4. Retinitis por CMV.
5. Ulcera corneal postraumatica.

43. Al explorar el campo visual, un escatoma centro-
cecal sugiere:

1. Esclerosis mltiple.
2. Neuritis ptica txica.
3. Glaucoma crnico.
4. Neuritis ptica isqumica.
5. Tumor cerebral.

44. En un paciente diagnosticada de Eritema Multi-
forme y afectacin conjuntival, con descenso de ni-
veles de Mucina en la secreccin lacrimal, encon-
traremos todo lo siguientes EXCEPTO :

1. Descenso del n de cl. caliciformes.
2. Tiempo de rotura de la pelcula lacrimal reducido.
3. Tincin patolgica con Rosa de Bengala.
4. Mejora con lgrimas artifiales.
5. Impregnacin de la tirilla secante (Prueba de
Schirmer) de 12 mm en 5 minutos.

45. En un RN con vtreo primario hiperplsico persis-
tente suele observarse todo lo siguiente EXCEPTO
:

1. Buftalmos al nacimiento.
2. Catarata.
3. DR.
4. Leucocoria.
5. Glaucoma.

46. En cuanto al cierre de ductus arterioso persistente
seale la respuesta correcta:

1. No debe realizarse en pacientes asintomaticos
2. La tasa de mortalidad es elevada
3. La hipertension pulmonar contraindica su cierre
4. En pacientes asintomaticos debe ligarse a 1-2 aos
de vida
5. El tratamiento medico es efectivo en el 50% de los
casos

47. Seale cual de los siguientes enunciados es inco-
rrecto con respecto al hijo de madre diabetica:

1. Tienen una mayor incidencia de S. de distress res-
piratorio
2. Se observa cardiomegalia en 1/3 casos
3. La incidencia de anomalias congenitas es 3 veces
mayor que en la poblacion general
4. Existen niveles plasmaticos elevados de hormona
de crecimiento en el feto
5. Son generalmente RN de peso elevado para la
edad gestacional

48. Cul NO es propia del Calostro?

1. Tiene ms grasas que la leche madura.
2. Tiene ms protenas que la leche madura.
3. Es ms rico en vitaminas que la leche madura.
4. Tiene mayor concentracin de Na+ y K+.
5. Contiene anticuerpos protectores.

49. Es FALSO en la Enf. de la membrana hialina:

1. Broncograma areo en la Rx de trax.
2. Inicio a las 6-12 h.
3. El fosfatidilglicerol suele estar aumentado.
4. Es ms frecuente cuanto menor peso tenga el RN.
5. Una de las complicaciones tpicas es el ducuts per-
sistente.

50. Las lesiones de la rama posterior de la arteria
cerebral media producen:

1. Afasia de conduccin.
2. S.de Wallenberg.
3. Amaurosis fugaz.
4. Afasia de Broca.
5. Incontinencia de esfnteres.

51. En un paciente en coma, cul de estos datos indi-
ca indemnidad del tronco de encfalo?

1. EESS en hiperextensin.
2. Respiracin de Biot.
3. Reflejo oculo-ceflico: desviacin conjugada en
sentido opuesto a la cabeza
4. Pupilas no reactivas.
5. Nistagmo de convergencia.

52. Un nio padece crisis de desconexin momentnea
con el entorno de no ms de 20 segundos de dura-
cin, tras las cuales no existe ninguna alteracin
neurolgica. Al evaluarlo, la hiperventilacin des-
encadena la crisis, apareciendo complejos punta-
onda a 3 Hz. Se desea efectuar un tratamiento en
rgimen de monoterapia que prevenga la apari-
cin de dichos fenmenos, pero tambin la even-
tualidad de otros fenmenos epilpticos, ya que los
padres cuentan una historia poco precisa de algn
episodio convulsivo en el nio que, aunque impre-
siona de convulsiones febriles, no queda claro dada
la escasa facilidad de los progenitores para expli-
carse. Qu frmaco utilizaras?

1. Etosuximida.
2. Valproico.
3. Trimetadiona.
4. Carbamacepina.
5. Fenobarbital.

53. Un paciente varn de 36 aos fue trasladado al
servicio de urgencias despus de sufrir un acceso
de cefalea aguda, seguido de cada brusca al suelo
y acompaado de vmitos. En las semanas previas
haba presentado tres episodios similares, en los
que, tras sentir un dolor intenso en regin frontal,
Centro de Estudios Aula Neptuno - GRANADA Ver.: 07/2000
www.aulaneptuno.com
- 6 -
caa desplomado al suelo. En ninguno de estos epi-
sodios haba llegado a perder el conocimiento. se-
gn relataba el mismo, junto con el dolor de cabe-
za, perda la fuerza en las piernas de forma repen-
tina, recuperndola al cabo de 1 2 minutos. Estos
episodios tenan relacin con maniobras de hiper-
extensin del cuello, como mirar hacia arriba o co-
ger un objeto de una estantera alta. En alguna
ocasin haba presentado dolor de cabeza intenso
al acostarse en decbito supino, que desapareca al
adoptar el decbito lateral. Desde haca unos me-
ses, el enfermo presentaba tambin dolor lumbar
sordo de moderada intensidad. Exploracin fsica:
en el momento del ingreso, el enfermo se encon-
traba obnubilado, con lenguaje pobre e incoheren-
te. Se detect un trastorno de la motilidad ocular
por parlisis de la supraversin. En el examen de
fondo de ojo, se apreci la existencia de un edema
de papila bilateral. La exploracin de pares cra-
neales no mostraba otras alteraciones. En cuanto
al sistema motor, se puso de manifiesto un marca-
do dficit de fuerza en ambas piernas con hiper-
rreflexia generalizada y signo de Babinski bilate-
ral. No presentaba rigidez de nuca ni otros signos
menngeos. En relacin al dolor lumbar, ste se in-
crementaba con la presin sobre las apfisis espi-
nosas lumbares. El signo de Lasgue era negativo
y no se encontraron signos de radiculopata. El re-
sto de la exploracin neurolgica y general fue
normal. Cul crees que es causa ms probable de
estos sntomas en el paciente en cuestin?

1. Crisis epilpticas atnicas del lbulo frontal.
2. Migraa vertebrobasilar.
3. Ataque de catapleja.
4. Hidrocefalia aguda intermitente.
5. Isquemia basilar.

54. Una mujer trae al hospital a su marido de 42 aos
a causa de una diplopa. El paciente despert hace
30 minutos y al ponerse los zapatos not que no
poda ver su pie izdo. En la exploracin fsica, el
paciente aparece ligeramente aletargado; el cuello
conserva su flexibilidad. Tiene un pulso de 82
l.p.m. y una presin arterial de 140/90 mm Hg. En
la exploracin ocular se observa un ligersima pto-
sis dcha. La pupila dcha es 1mm mayor que la iz-
da, y ambas reaccionan a la luz. Los fondos de ojo
son normales. El paciente tiene una hemianopsia
homnima izda. Mientras est explorando sus re-
flejos, el paciente est cada vez ms adormilado y
obnubilado. Presenta una hemiparesia izda. Al
volver a examinar sus ojos comprueba que el ojo
dcho se desva lateralmente. El ojo izdo muestra
movimientos bruscos en ojo de mueca, hacia
arriba, hacia abajo o medialmente. El paciente se
va poniendo rgido, con temblores musculares di-
fusos. Qu medida conviene adoptar a continua-
cin?:

1. Comprobar los reflejos oculovestibulares con agua
fra.
2. Solicitar inmediatamente una tomografa compu-
tadorizada cerebral.
3. Realizar una puncin lumbar.
4. Intubar, hiperventilar y administrar manitol por va
intravenosa.
5. Administrar oxgeno al 40% por va nasal; poste-
riormente, completar la exploracin neurolgica.

55. Un varn de 42 aos, que en las ltimas fechas ha
tenido dificultad para concentrarse en su trabajo,
solicita atencin mdica debido a movimientos
irregulares en sacudidas de las extremidades y los
dedos. Una hermana y un to murieron en institu-
ciones mentales, y su madre present demencia en
edades medias. El dgto.ms probable es:

1. Degeneracin cerebral alcohlica.
2. Corea de Huntington.
3. Enfermedad de Wilson.
4. Enf.de Hallervorden Spatz.
5. Enf.de Gilles de la Tourette.

56. En relacin con la ataxia de Friedreich, es falso:

1. La clnica comienza antes del nacimiento por una
reduccin en los movimientos fetales.
2. Hay una prdida de neuronas de los ganglios de la
raz dorsal y degeneracin de los cordones
posteriores.
3. Pese a haber arreflexia, tiene Babinski (+).
4. La ataxia es predominantemente sensitiva.
5. No suele haber retraso mental.

57. En el diagnstico diferencial de las demencias
existen datos propios de algunas de ellas. Una de
las que se mencionan a continuacin no es cierta a
este respecto:

1. La atrofia corticobasal o degeneracin corticoden-
tadongrica presenta distona y mioclonas de una
mano, sndrome de la mano aliengena, apraxia y
oftalmopleja supranuclear.
2. Se denomina demencia cortical posterior a un
cuadro caracterizado por dificultades en la ejecu-
cin de actos complejos y desorientacin espacial
con conservacin casi completa del lenguaje y la
memoria.
3. La demencia del corea de Huntington suele ser de
tipo subcortical, es decir, con relativa preservacin
de las fasias, praxias y gnosias.
4. La demencia de Pick afecta casi exclusivamente a
hombres y en ms del 90% de los casos tiene his-
toria familiar positiva.
5. Las alteraciones cromosmicas en la Demencia de
Alzheimer se han localizado en el cromosoma
21q, donde se halla el gen de la protena precurso-
ra de amiloide (PPA) y en el 14q, ocasionndose
en este caso fenotipos ms graves que en el ante-
rior.

58. Paciente de 17 aos que acude a consulta por cua-
dro de ataxia de la marcha de unos dos meses de
evolucin,disartria, hipopalestesia con arre-
flexia,paraparesia con reflejos cutneoplantares
extensores. Explorando al muchacho se aprecia
que la ataxia presenta caractersticas cerebelosas y
Centro de Estudios Aula Neptuno - GRANADA Ver.: 07/2000
www.aulaneptuno.com
- 7 -
escoliosis.En el estudio complementario se apre-
cian valores altos de glucemia y transtornos tem-
pranos en la repolarizacin con ecocardiograma
con evidencia de miocardiopata.Tras realizar una
TC craneal se aprecia atrofia cerebelosa y dilata-
cin de IV ventrculo. Tras llegar con estos datos a
un diagnstico de presuncin,qu afirmacin con-
siderara correcta:

1. Se trata de una Enf. de Kugelberg-Welander.
2. Con los datos expuestos no se puede hacer un
diagnsdtico diferencial entre los sindromes
espinocerebelosos.
3. Su tratamiento seran anticolinrgicos.
4. Este cuadro no tiene tratamiento.
5. Se trata de una enfermedad de motoneurona del
adolescente.

59. Un adolescente presenta ceguera unilateral, cefa-
lea, vrtigo, nistagmo direccional, diplopia y ataxia
de la marcha. A la exploracin no se comprueba la
existencia de lesiones cutneas. Cul de los si-
guientes cuadros puede sospecharse?

1. Sndrome de Klipel Trenaunay.
2. Enfermedad de Von Hippel Lindau.
3. Enfermedad de Sturge Weber.
4. Esclerosis tuberosa.
5. Hipomelanosis de Ito.

60. La forma de presentacin ms frecuente del Gui-
llain-Barr es:

1. Parlisis facial.
2. Dolores musculares en EEII.
3. Parestesias en manos.
4. Debilidad en los msculos de la mano.
5. Debilidad muscular en EEII.

61. No es manifestacin del sndrome de la cauda
equina:

1. Paraparesia espstica asimtrica.
2. Dficit sensitivo "en silla de montar".
3. Prdida de reflejo rotuliano.
4. Disfuncin vesical.
5. Prdida de reflejo aquleo.

62. Tras un proceso infeccioso de vas respiratorias
altas, un paciente comienza a quejarse de dolor en
la espalda, parestesias y adormecimiento en las ex-
tremidades inferiores. Varios das despus nota
tambin prdida de fuerza en las piernas con in-
continencia de esfnteres. Se observa en la explora-
cin de las extremidades inferiores una espastici-
dad con hiperreflexia osteotendinosa, as como un
nivel sensitivo a nivel dorsal bajo. En el LCR hay
100 linfocitos/mm3 y 40 mg/dl de protenas. La ra-
diografa de columna y la mielografa son norma-
les. El diagnstico ms probable es:

1. Absceso epidural espinal.
2. Ependimoma.
3. Polirradiculoneuritis.
4. Hematomielia.
5. Mielitis transversa aguda.

63. Una mujer de 40 aos acude a la consulta con un
intenssimo dolor en el lado derecho de la cara
(mejilla, nariz, labios y zona mandibular). Desde
hace unos dos meses viene padeciendo dicho dolor
en crisis a lo largo de todo el da, con escasos perio-
dos libres de sintomatologa entre ellas. El dolor
apareca al tocar la nariz o el labio superior, o al
hablar, masticar y tragar. En la anterior paciente
es de eleccin el siguiente frmaco:

1. Carbonato de litio.
2. Diclofenac.
3. Carbamacepina.
4. Paracetamol.
5. Derivados morfnicos.

64. Un paciente con SIDA y en tratamiento con AZT
desarrolla debilidad muscular proximal con mial-
gias. La CPK es normal. En la biopsia muscular se
observan fibras deshilachadas perifasciculares ro-
jo desigual. Qu es ms probable que padezca?

1. Distrofia muscular.
2. Polimiositis idioptica.
3. Miopata por AZT.
4. Miopata nemalnica.
5. Vasculitis microscpica.

65. Indica la proposicin errnea sobre las distintas
formas de amnesia:

1. La amnesia temporal es una caracterstica frecuen-
te del traumatismo craneoenceflico; se produce
amnesia retrgrada, sobre todo del sistema de
memoria inmediata, y amnesia antergrada o alte-
racin en el aprendizaje de nueva informacin.
2. La amnesia global transitoria es un sndrome en el
que una persona presenta de forma sbita confu-
sin y amnesia estando previamente bien.
3. La amnesia selectiva para la identidad personal
puede formar parte de los trastornos disociativos
de fuga psicgena.
4. Los pacientes con Sndrome de Korsakoff presen-
tan una memoria inmediata conservada, pero son
incapaces de retener informacin adquirida ms
all del periodo de duracin de dicha memoria.
5. Aunque no se conoce de forma exhaustiva la base
anatmica de la memoria, parece claro que sta se
relaciona con los lbulos parietales, ya que una le-
sin bilateral de stos puede producir amnesia.

66. En la encefalopata hipercpnica no se suele pro-
ducir:

1. Asterixis.
2. Lesin cerebral irreversible.
3. Papiledema.
4. Cefalea generalizada, occipital o frontal.
5. Indiferencia y falta de atencin respecto al am-
biente.

Centro de Estudios Aula Neptuno - GRANADA Ver.: 07/2000
www.aulaneptuno.com
- 8 -
67. Para la decisin de administrar tratamiento far-
macolgico y la eleccin del mismo en la Enferme-
dad de Parkinson es til recurrir a la escala de
Hoehn y Yahr, que define cinco estadios. En la En-
fermedad leve (estadios 1 y 2) la mayora de los cl-
nicos recomiendan administrar deprenilo, un in-
hibidor selectivo de la MAO-B, mientras que
cuando la incapacidad es importante (estadios 3 y
superiores) es necesario instaurar la administra-
cin de levodopa. Despus de este publirreportaje,
y como hace falta preguntar algo: cul de los si-
guientes estadios de Hoehn y Yahr corresponde al
grado 3?

1. Afectacin bilateral con desequilibrio postural le-
ve; el paciente puede llevar una vida independien-
te.
2. Afectacin unilateral.
3. Afectacin bilateral sin alteraciones posturales.
4. Afectacin bilateral con inestabilidad postural; el
paciente requiere una ayuda considerable.
5. Afectacin intensa y plenamente establecida; el
paciente permanece en una cama o una silla de
ruedas.

68. Seala cul de las siguientes contiene la triada de la
hidrocefalia normotensiva:

1. Deterioro intelectual, trastorno de la marcha e in-
continencia de esfnteres.
2. Deterioro intelectual, ataxia y mioclonas.
3. Deterioro intelectual, trastorno de la marcha y
mioclonas.
4. Deterioro intelectual, ataxia e incontinencia esfin-
teriana.
5. Deterioro intelectual, piramidalismo y mioclonas.

69. La presencia de dolor otomastoideo con dolor
facial y debilidad del recto externo homolateral, es
indicativo de trombosis de una vena o seno cere-
bral. Selalo:

1. Seno cavernoso.
2. Seno longitudinal superior.
3. Trombosis de vena cortical.
4. Trombosis de seno lateral.
5. Trombosis de seno recto.

70. Cul de estos efectos beneficiosos puede estar
producido por la disminucin del 2-3 DPG intrae-
ritrocitario?

1. Reduccin de la afinidad de la Hb por el O2.
2. Mayor cesin de O2 a tejidos.
3. Desplazamiento a la derecha de la curva de satu-
racin de Hb.
4. Mayor captacin de O2 por el capilar pulmonar en
ciertas circunstancias (altitud).
5. Elevacin de la P50.

71. Sobre los mecanismos de hipoxemia e hipercapnia,
indica la falsa:

1. La causa ms frecuente de hipoxemia son los des-
equilibrios V/Q.
2. El mecanismo esencial que subyace a todos los
casos de hipercapnia es la inadecuada ventilacin
alveolar para la cantidad de CO2 producido.
3. En la hipoventilacin el gradiente alveolo arterial
de O2 est normal.
4. La hipoxemia causada predominantemente por
cortocircuito (shunt) es refractaria al tratamiento
con oxgeno.
5. El efecto shunt cursa con gradiente alveolo arterial
de oxgeno normal.

72. El diagnstico del asma debe establecerse inicial-
mente con el estudio del VEMS antes y despus de
inhalar:

1. Histamina.
2. Metacolina.
3. Hiperventilacin de aire fro.
4. Agonistas beta adrenrgicos.
5. Metilxantinas.

73. Un paciente de 60 aos, con enfisema, es traslada-
do a un servicio de urgencias por una ambulancia
del SEDU, en la cual se le ha administrado oxige-
noterapia en mascarilla. Hace tres das observ
que su esputo haba cambiado de color y aumen-
tado en cantidad. Su esposa llam a la ambulancia
cuando el paciente present disnea y confusin re-
pentinas. Al llegar al hospital se observa somnolen-
cia. En la exploracin se comprueban crepitantes
hasta campos medios y sibilantes; hay edemas pe-
rifricos y ascitis notorios. Analtica: Hb: 18 g./dL;
Gases arteriales: pH: 7,08; PO2: 148 mm.Hg.;
PCO2: 106 mm.Hg. El tratamiento inmediato ms
apropiado para el paciente descrito es:

1. Bicarbonato i.v.
2. Intubacin endotraqueal y ventilacin asistida.
3. .Administracin de terbutalina mediante nebuliza-
dor con compresor de aire.
4. Suspender el oxgeno suplementario.
5. Adrenalina subcutnea.

74. En este enfermo, las manifestaciones de fallo car-
diaco derecho deben tratarse con:

1. Diazxido.
2. Digoxina.
3. Hidralacina.
4. Oxgeno.
5. Flebotoma.

75. En cul de los siguientes cuadros clnicos no se
deben administrar pequeas dosis de heparina
subcutnea para prevenir las tromboembolias?

1. Durante la recuperacin de un IAM.
2. Tras una laparotoma en la que se ha descubierto
un carcinoma de colon que no se puede resecar.
3. En un hombre obeso que est encamado con una
insuficiencia cardiaca grave.
4. Tras una apendicectoma de urgencia a una mujer
con antecedentes previos de trombosis venosas
Centro de Estudios Aula Neptuno - GRANADA Ver.: 07/2000
www.aulaneptuno.com
- 9 -
profundas.
5. Tras la extirpacin quirrgica de una catarata a un
hombre mayor.

76. Afirmacin falsa en las neumonas:

1. La positividad del hemocultivo es de un 30%.
2. La neumona intrahospitalaria suele ser polimicro-
biana.
3. La faringitis previa hace pensar en el micoplasma
como agente responsable.
4. Citomegalovirus suele provocar una bronconeu-
mona.
5. La asociacin de esplenomegalia lleva a pensar en
la fiebre Q.

77. Si en un paciente con hipertensin arterial pulmo-
nar la radiografa muestra campos pulmonares
claros, las pruebas funcionales orientan hacia un
patrn restrictivo y en la gasometra arterial se
comprueba hipercapnia, nuestra principal sospe-
cha debe ser:

1. Sndromes de hipoventilacin.
2. Insuficiencia cardiaca derecha.
3. Insuficiencia cardiaca izquierda.
4. Embolismos pulmonares de repeticin.
5. Hipertensin pulmonar primaria.

78. Enfermo de 48 aos de edad, sin antecedentes
personales ni familiares de inters. No fumador.
Intervenido quirrgicamente para efectuar artro-
desis lumbosacra con placa de Louis por espondi-
lolistesis LV-SI. Permaneci en reposo absoluto 9
das y fue dado de alta en el hospital para guardar
un periodo de concalecencia en su domicilio. 24
horas despus consulta de urgencia por presentar
dolor agudo en costado izquierdo de instauracin
rpida, continuo, irradiado a regin escapular. Se
acenta con los movimientos respiratorios y la tos
y se acompaa de ligera disnea. Exploracin: Ta-
quipnea y taquicardia, disminucin del murmullo
vesicular en base pulmonar izquierda. Extremida-
des sin signos de tromboflebitis ni trombosis. Ga-
sometra: PO2 47 mm.Hg., PCO2 36 mm.Hg.
Rx.de trax: Velamiento del seno costodiafragm-
tico izquierdo. La primera prueba que indicaras
para confirmar el dgto.de sospecha es:

1. Estudio hemodinmico pulmonar.
2. Ecocardiografa.
3. Gammagrafa pulmonar.
4. Fibrobroncoscopia con biopsia.
5. Toracocentesis y biopsia pleural.

79. Una paciente de 50 aos lleva varios meses notan-
do una molesta tos seca y fatigndose cada vez ms
con tareas que antes realizaba sin disnea. No tiene
antecedentes de inters y en su familia nadie ha
padecido "de los bronquios". A la exploracin hay
crepitantes teleinspiratorios en ambas bases pul-
monares. La radiografa de trax demuestra un
patrn en vidrio deslustrado bilateral de predomi-
nio inferior y la gasometra revela hipoxemia e
hipocapnia. La analtica es normal y las determi-
naciones inmunolgicas sricas son negativas. No
hay disfuncin del corazn izquierdo. En la espi-
rometra forzada existe una reduccin de los
volmenes pulmonares y de la DLCO. Qu
entidad representa el diagnstico ms probable?

1. Alveolitis fibrosante criptogentica.
2. Asbestosis pulmonar.
3. Enfermedad intersticial en paciente con lupus eri-
tematoso.
4. Adenocarcinoma bronquioloalveolar.
5. Histiocitosis de clulas de Langerhans.

80. Paciente joven de constitucin astnica que presen-
ta dolor brusco en p. de costado y dis-
nea.Progresivamente asfixia, cianosis e ingurgit
yugular. Diagnstico de sospecha.

1. IAM.
2. IM aguda.
3. TEP.
4. NT hipertensivo.
5. Neumomediastino.

81. Indique la causa ms frecuente de trasudado:

1. Cirrosis heptica.
2. Insuficiencia cardaca congestiva.
3. Sndrome nefrtico.
4. Sndrome de MEIGS.
5. Pericarditis constrictiva.

82. Cul de las siguientes afirmaciones es cierta res-
pecto a la Enfermedad Venoclusiva pulmonar? :

1. Su etiologa es vrica.
2. Crusa con HT pulmonar prostcapilar.
3. Su tto. especfico son los glucocorticoides.
4. La Rx de trax suele ser normal.
5. Est causada por la obstruccin de las arteriolas
pulmonares.

83. De las siguientes imgenes radiolgicas, slo una
no concuerda con el diagnstico de cncer bronco-
pulmonar:

1. Imagen derrame-atelectasia.
2. Ensanchamiento hiliar.
3. Opacidad velada de un vrtice pulmonar.
4. Ndulo pulmonar solitario con calcificaciones en
su interior.
5. Opacidad cavitada con paredes irregulares y
muesca.

84. Paciente con sospecha de padecer un cncer bron-
copulmonar. A la hora de establecer el diagnstico
etiolgico, y antes de conocer los resultados de la
biopsia; cul de las siguientes combinaciones de
hallazgos nos hara sospechar que nos encontra-
mos ante un adenocarcinoma?

1. Ataxia y vrtigos. TFM. Ginecomastia. Calcio:
8.9mg./dL. Sodio: 128mEq./L. ACTH: 48pg./mL.
Centro de Estudios Aula Neptuno - GRANADA Ver.: 07/2000
www.aulaneptuno.com
- 10 -
2. Ginecomastia. S.miasteniforme. Osteopata hiper-
trfica numica. ACTH: 60pg./mL. Ca++:
13.7mg/dL. Na+: 136mEq./L.
3. Acantosis nigricans. Ginecomastia. TFM. Osteo-
pata hipertrfica. Na+: 138mEq/L. Ca++:
8.7mg./dL. ACTH: 32pg./mL.
4. Dolor en hombro derecho. Acantosis nigricans.
Dolor y parestesias distales. ACTH: 240pg/mL.
Calcio: 10.9mg./dL. Sodio: 119mEq/L.
5. TFM. Ginecomastia. Cavitacin. Acantosis nigri-
cans. S. miasteniforme. K+: 3.1mEq./L. Sodio:
151mEq/L. Calcio: 9.3mg/dL.

85. Paciente varn de 49 aos diagnosticado de glo-
merulonefritis crnica, en tratamiento con hemo-
dilisis desde 2 aos antes. En las tres ltimas se-
manas inici fiebre de hasta 38,6C, dolor pleurti-
co en costado izquierdo y tos no productiva. Se
diagnostic de derrame pleural que corresponda
a un exudado de predominio linfocitario sin clulas
malignas. La biopsia pleural con aguja mostr
pleuritis crnica inespecfica. En la exploracin f-
sica destacaba, en la auscultacin pulmonar, hipo-
fonesis y roce pleural en la mitad inferior del hemi-
trax izquierdo, con matidez a la percusin y dis-
minucin de las vibraciones vocales. No hubo otros
hallazgos de inters. El hemograma mostraba una
ligera anemia, pero no haba alteraciones en la
bioqumica srica ni en la coagulacin. No se detec-
taron grmenes en el lquido pleural (cultivo con-
vencional y LOWENSTEIN) ni en el material de
la biopsia. Los anticuerpos antinucleares, latex,
WAALER-ROSE y complemento fueron negati-
vos o normales. En la eco-cardiografa se objetiv
ligero derrame pericrdico. En la radiografa de
trax se observaba una imagen compatible con
derrame pleural izquierdo que ocupaba la mitad
del hemitrax, sin adenopatas, cardiomegalia ni
otras alteraciones valorables. Diagnstico MAS
probable:

1. Derrame pleural neoplsico.
2. Insuficiencia cardaca.
3. Derrame pleural urmico.
4. Derrame pleural tuberculoso.
5. Sndrome nefrtico.

86. El sustrato anatomo patolgico de la afectacin
respiratoria en el contexto de la enfermedad injer-
to contra husped es (it is):

1. Alveolitis.
2. Vasculitis.
3. Bronquiolitis.
4. Traquetis.
5. Bronquitis.

87. Sobre la patogenia de las neumonitis por hipersen-
sibilidad (alveolitis alrgicas extrnsecas) indica la
respuesta que consideres incorrecta:

1. La lesin inicial se caracteriza por un incremento
de polimorfonucleares en los alveolos y pequeas
vas respiratorias.
2. Esta lesin inicial se sigue de la formacin de in-
munocomplejos responsables del desarrollo de
granulomas.
3. Los pacientes con exposicin muy reciente pue-
den mostrar un aumento de linfocitos T CD4 en el
lavado broncoalveolar.
4. Los pacientes con exposicin reciente o repetida al
antgeno muestran un aumento de polimorfonu-
cleares en el lquido del lavado.
5. En la mayora de los pacientes examinados tras la
recuperacin de una enfermedad aguda, los linfo-
citos T del lquido del lavado pertenecen predomi-
nantemente al subgrupo CD8.

88. Si un paciente con bronquitis de repeticin y oca-
sionales hemoptisis presenta en la radiografa de
trax una masa slida en lbulo inferior izquierdo
con vasos que festonean su periferia, el diagnstico
ms probable es:

1. Mieloma mltiple.
2. Adenocarcinoma bronquioloalveolar.
3. Secuestro pulmonar.
4. Sndrome de la cimitarra.
5. Bronquiectasias saculares.

89. Cul de los siguientes hallazgos se observa en la
tomografa computorizada torcica en la aspergi-
losis broncopulmonar alrgica?

1. Vidrio deslustrado.
2. Bronquiectasias centrales.
3. Quistes areos difusos.
4. Ndulos pulmonares.
5. Condensaciones alveolares parcheadas.

90. No es una lesin tpica de la artritis reumatoide:

1. Tendosinovitis.
2. Infiltrado perivascular por linfocitos.
3. Esclerosis subcondral.
4. Erosiones seas.
5. Granulomas con fibrina e histiocitos.

91. Qu afirmacin es falsa sobre la granulomatosis
de Wegener?

1. Los ANCA-c con especificidad anti proteinasa 3
son muy especficos de esta enfermedad, pero slo
se elevan en el 50% de los pacientes con enferme-
dad activa generalizada.
2. La afectacin renal es ms frecuente que la cut-
nea.
3. El seguimiento de la enfermedad en los pacientes
tratados se realiza por las manifestaciones clnicas,
la VSG y los ANCA-c.
4. En las vas respiratorias y el parnquima pulmonar
se observa la presencia de granulomas necrosantes
con abundantes clulas gigantes multinucleadas y
vasculitis necrosante que afecta pequeas arterias
y venas.
5. La afectacin renal suele consistir en una glomeru-
lonefritis necrosante segmentaria y focal.

Centro de Estudios Aula Neptuno - GRANADA Ver.: 07/2000
www.aulaneptuno.com
- 11 -
92. Un joven presenta prdida de 6 kg.de peso, mial-
gias, dolor testicular, hipertensin arterial diastli-
ca y elevacin de las cifras de urea y creatinina.
Existe una livedo reticularis a la exploracin. La
VSG est acelerada y el factor reumatoide y los
ANA son negativos. La biopsia cutnea se informa
como normal. Cul es el siguiente paso diagnsti-
co?

1. Arteriografa renal.
2. Repetir la biopsia cutnea.
3. Repetir la determinacin de FR y ANA.
4. Biopsia renal.
5. Estudio del lquido sinovial.

93. Cul es la nica vasculitis en la que se ha descrito
asociacin significativa con algn antgeno del
HLA?

1. Enfermedad de Kawasaki.
2. Poliarteritis microscpica.
3. Enfermedad de Behet.
4. Granulomatosis de Wegener.
5. Vasculitis aislada del sistema nervioso central.

94. Respecto a las artritis virales, seala la incorrecta:

1. Generalmente se afectan varias articulaciones y
los signos inflamatorios son menores que en las
bacterianas.
2. Entre los virus ms frecuentes estn la rubeola y el
virus de la hepatitis B.
3. Se ha descrito asociada al VIH.
4. El tratamiento exige artrocentesis evacuadora si
hay signos inflamatorios en la articulacin.
5. Est indicado el reposo articular y la administra-
cin de AINEs.

95. En un nio con extremidades largas y finas, arac-
nodactilia, pectus excavatum, raz de la aorta dila-
tada y pies planos, cul de las siguientes manio-
bras diagnsticas elegiras para confirmar el dia-
gnstico?

1. Electroencefalograma.
2. RMN craneal.
3. Medicin del cobre srico.
4. Gammagrafa sea.
5. Examen oftalmolgico con lmpara de hendidura.

96. Si en un paciente se diagnostica una condrocalci-
nosis, cul ser el diagnstico etiolgico ms
frecuente?

1. Condrocalcinosis familiar.
2. Hiperparatiroidismo primario.
3. Hemocromatosis.
4. Hipomagnesemia.
5. Condrocalcinosis idioptica.

97. Varn de 35 aos acude a urgencias con intenso
dolor en su tobillo izquierdo, que est inflamado,
rojo y caliente. Niega traumatismo previo. La ar-
trocentesis demuestra cristales intracelulares con
birrefringencia en el lquido sinovial, y el Gram re-
sulta negativo. Cul sera tu tto.inicial ms correc-
to?

1. Alopurinol 300 mg/da.
2. Prednisona, 5 mg, cuatro veces al da.
3. Indometacina, 75 mg., tres veces al da.
4. Probenecid, 500 mg, dos veces al da.
5. Penicilina procana, 1.200.000 uu.en dosis nica.

98. En cul de los siguientes casos es ms probable
que sea correcto el diagnstico de espondilitis an-
quilopoytica?

1. Un hombre de 28 aos que, en los ltimos 10 aos
ha tenido dolor y rigidez en la parte inferior de la
espalda, que empeora por la noche y se alivia con
la actividad.
2. Un hombre de 32 aos que en los ltimos 5 aos
ha tenido dolor en la parte baja de la espalda que
empeora al moverse pero se alivia con el reposo
en cama.
3. Un hombre de 34 aos que en los ltimos 10 aos
ha tenido episodios intermitentes de dolor leve en
la parte baja de la espalda; actualmente sin embar-
go y de forma brusca, es incapaz de realizar la dor-
siflexin del dedo gordo del pie derecho.
4. Un hombre de 65 aos que en los ltimos 10 aos
ha tenido dolor en la parte baja de la espalda, irra-
diado hacia abajo por la parte posterior de ambos
muslos, hasta las rodillas.
5. Un hombre de 72 aos que, desde hace 15 aos,
ha tenido dolor progresivo en la espalda que em-
peora al caminar, pero mejora en reposo y al incli-
narse hacia adelante.

99. Respecto al diagnstico de la Artritis Reumatoide
indica la afirmacin errnea:

1. La presencia de factor reumatoide no establece el
diagnstico aunque puede ser importante de cara
al pronstico, ya que pacientes con mayores ttulos
del mismo suelen presentar una afectacin ms
grave y progresiva con manifestaciones extraarti-
culares.

2. El factor reumatoide est presenta en ms del 65%
de pacientes con AR y en un 5% de la poblacin
general.
3. La radiografa de las articulaciones afectadas no es
til para establecer el diagnstico en las primeras
fases de la enfermedad, pues slo se observa tu-
mefaccin de partes blandas.
4. Las alteraciones radiolgicas, por lo anteriormente
expuesto, no se incluyen dentro de los criterios de
la ARA para la Artritis Reumatoide.
5. Las caractersticas del lquido sinovial obtenido
por puncin no se incluyen dentro de los criterios
de la ARA para el diagnstico de Artritis Reuma-
toide.

100. Cul es falsa en el tratamiento de la sarcoidosis?

1. Para el tratamiento de la sarcoidosis pulmonar con
Centro de Estudios Aula Neptuno - GRANADA Ver.: 07/2000
www.aulaneptuno.com
- 12 -
disnea de esfuerzos moderados-pequeos deben
emplearse los corticoides en aerosol, reservando la
va oral para los casos que no respondan.
2. A no ser que la clnica pulmonar sea muy impor-
tante, se recomienda vigilancia sin tratamiento du-
rante 2-3 meses, aunque otros autores defienden el
inicio del mismo en cuanto haya cambios paren-
quimatosos en la radiografa de trax.
3. En el caso de la afectacin ocular, la decisin de
iniciar tratamiento se basa en el examen con lm-
para de hendidura y las pruebas de agudeza visual.
4. En el caso del corazn y del Sistema Nervioso
Central la decisin se basa en la gravedad de las
lesiones; los pacientes con disfuncin leve pueden
ser vigilados sin tratamiento, en tanto aquellos pa-
cientes con alteraciones importantes deben recibir
tratamiento.
5. La ciclosporina es ineficaz en las manifestaciones
pulmonares, aunque algunas publicaciones sugie-
ren que puede ser til en las formas extrapulmona-
res resistentes a los corticoides.

101. EN cul de los siguientes pacientes esperaras
encontrar positividad para los anticuerpos anti-
histona?

1. Un hombre de 60 aos con una erupcin, artritis y
pleuritis que aparecieron tras realizar un tratamien-
to anti hipertensivo con hidralacina.
2. Una joven con historia de nefritis adquirida al dar
a luz un nio con bloqueo cardiaco congnito.
3. Una mujer de 37 aos con historia de trombosis
arteriales recidivantes a quien se encuentra un
tiempo de tromboplastina parcial prolongado y un
resultado falso positivo de Venereal Disease Re-
search Laboratory.
4. Una mujer joven con un sndrome CREST (calci-
nosis, fenmeno de Raynaud, hipomotilidad eso-
fgica, esclerodactilia y telangiectasias).
5. Una mujer de 41 aos con dolores en la pared an-
terior del trax en quien se diagnostica un sndro-
me SAPHO (sinovitis, acn, pustulosis, hiperosto-
sis, ostetis).

102. Sobre la artropata neuroptica, una de las siguien-
tes afirmaciones es falsa:

1. Son causas habituales en los adultos la siringomie-
lia, la mielosis funicular y los traumatismos medu-
lares, entre otros.
2. La diabetes es la causa ms frecuente y suele afec-
tar a los huesos del tarso, articulaciones tarso-
metatarsianas y metatarso-falngicas.
3. EL tratamiento adecuado de la enfermedad neuro-
lgica tiene muy poca o nula influencia sobre la
artropata, que progresa con el tiempo.
4. La alteracin radiogrfica ms caracterstica con-
siste en la observacin de un doble, triple o cu-
druple contorno seo.
5. En los estadios finales hay una alteracin grotesca
de la morfologa articular y una acentuada hiper-
motilidad.

103. Una de las siguientes asociaciones entre vasculitis y
datos caractersticos de las mismas es errnea. In-
dcala:

1. En la Granulomatosis de Wegener hay una espe-
cial susceptibilidad a las infecciones por bacilos
gram negativos.
2. La patergia es un dato de gran importancia para el
diagnstico de la Enfermedad de Behet.
3. En la Arteritis de Horton, tanto el factor reumatoi-
de como los ANA suelen ser negativos; es tpico,
por otra parte, que la VSG est muy acelerada.
4. Es frecuente que los nios con Sndrome de Ka-
wasaky presenten una trombocitosis superior a
500.000 clulas/mm3.
5. En la poliarteritis nodosa las lesiones anatomo pa-
tolgicas estn en distintas fases de evolucin.

104. Un paciente presenta una presin sistlica de 130
mm.Hg. en brazo derecho y de 70 mm.Hg. en bra-
zo izquierdo. Probablemente tendr:

1. Estenosis de subclavia izquierda.
2. Estenosis de subclavia derecha.
3. Oclusin de subclavia derecha.
4. Oclusin de subclavia izquierda.
5. Ninguna de las anteriores.

105. En las siguientes enfermedades puede estar indi-
cado el trasplante pulmonar unilateral, excepto:

1. Mucoviscidosis.
2. Linfangiomiomatosis.
3. Hipertensin pulmonar primaria.
4. Hipertensin pulmonar por Eisenmenger.
5. Obstruccin crnica al flujo areo.

106. En la radiografa simple de trax, la presencia de
uno de estos signos no se asocia con rotura artica,
tras un traumatismo torcico cerrado:

1. Enfisema mediastnico.
2. Borramiento del cayado artico.
3. Desplazamiento traqueal.
4. Derrame pleural.
5. Ensanchamiento de la lnea paratraqueal derecha.

107. En un nio de 20 meses de edad se sienta indica-
cin quirrgica por sospecha de apendicitis. Al
efectuar la laparotoma se comprueba que el
apndice es normal y no hay tampoco inflamacin
de los ganglios mesentricos. El leon terminal
tambin tiene un aspecto normal. Qu entidad
debemos sospechar en siguiente lugar?

1. Invaginacin intestinal.
2. Atresia ileal.
3. Diverticulitis de Meckel.
4. Diverticulitis colnica.
5. Enfermedad de Hirchsprung.

108. La causa ms frecuente de isquemia aguda mesen-
trica es:

1. Arteriosclerosis mesentrica avanzada.
Centro de Estudios Aula Neptuno - GRANADA Ver.: 07/2000
www.aulaneptuno.com
- 13 -
2. Infarto mesentrico no oclusivo.
3. Embolia mesentrica.
4. Trombosis venosa mesentrica.
5. Panarteritis nodosa.

109. Un paciente de 38 aos sin antecedentes consult
por dolores torcicos de tipo anginoso de varias
semanas de evolucin. El ECG era normal y en la
radiografa de trax se comprobaba un notable
ensanchamiento del botn artico con imgenes
calcificadas. La ecocardiografa transesofgica
confirm la presencia de un aneurisma de la aorta
torcica descendente de unos 7 cm.de dimetro.
Qu actitud es la ms correcta?

1. Reseccin del saco aneurismtico y sustitucin
por prtesis sinttica de Dacron.
2. Control peridico cada seis meses por ECO trans-
esofgica.
3. By pass laterolateral mediante interposicin de sa-
fena.
4. Reseccin y anastomosis terminoterminal.
5. Tratamiento con nitroprusiato en UCI y reevalua-
cin a los 5 das.

110. Cul es la causa que justifica la intervencin qui-
rrgica en los aneurismas de las arterias poplteas,
aunque sean de pequeo tamao?

1. Que se rompen ms frecuentemente que los aneu-
rismas de los grandes vasos.
2. Que si se rompen es ms difcil diagnosticarlos
que los de los grandes vasos y por eso conllevan
mayor mortalidad.
3. Que suelen ser bilaterales con mucha frecuencia.
4. Que la trombosis del saco con embolia distal es
mucho ms frecuente que en los aneurismas arti-
cos.
5. Que son ms fciles de operar.

111. Durante cunto tiempo tiene efecto negativo en la
agregacin plaquetaria el cido acetilsaliclico ad-
ministrado un cierto da?

1. Uno a dos das.
2. Tres a cuatro das.
3. Siete a nueve das.
4. Dos a tres semanas.
5. Durante varias generaciones porque es como una
gran cadena y permite contar al nio por las no-
ches el cuento del osito Winnie Pu.

112. Ante un politraumatizado, una de las siguientes
lesiones es una prioridad quirrgica absoluta:

1. Vollet costal.
2. Neumoperitoneo.
3. Conmocin cerebral.
4. Fractura de la base del crneo.
5. Rotura de bazo por estallido.

113. Es falso al referirnos a las hernias inguinales direc-
tas que:

1. Su origen posterior est a nivel de la fosita inguinal
media.
2. Con cierta frecuencia pueden ser bilaterales.
3. No suelen estrangularse.
4. Se originan por una debilidad en la fascia transver-
salis.
5. Casi siempre aparecen en mujeres.

114. Entre las siguientes enfermedades, figura una que
es causa de hipertiroidismo secundario, y hay que
tenerla en cuenta en el diagnstico diferencial con
el adenoma hipofisario productor de TSH. Sela-
la.

1. Sndrome de Refetoff.
2. Resistencia hipofisaria a las hormonas tiroideas.
3. Sndrome del enfermo eutiroideo.
4. Hipertiroxinemia familiar disalbuminmica.
5. Presencia de anticuerpos anti T4.

115. Respecto a las acciones de la hormona del creci-
miento, slo una de las siguientes definiciones es
correcta en su totalidad:

1. Promueve el crecimiento seo longitudinal, favo-
rece la proteolisis muscular, la lipognesis en tejido
graso, es antiinsulnica y aumenta la eficacia del
sistema inmune.
2. Promueve el crecimiento seo longitudinal, esti-
mula la sntesis proteica muscular, la lipolisis en el
tejido adiposo, es antiinsulnica y aumenta la efi-
cacia del sistema inmune.
3. Promueve el crecimiento seo longitudinal, esti-
mula la sntesis proteica muscular, la lipognesis
en tejido adiposo, tiene accin hipoglucemiante y
aumenta la eficacia del sistema inmune.
4. Promueve el crecimiento seo longitudinal, esti-
mula la sntesis proteica muscular, la lipolisis en te-
jido adiposo, tiene accin hipoglucemiante y au-
menta la eficacia del sistema inmune.
5. Inhibe el crecimiento del hueso, los cartlagos, los
msculos y la capacidad econmica del individuo.

116. Indica la respuesta falsa sobre la regulacin del eje
lactotropo y las acciones de la prolactina:

1. La prolactina es la unica hormona hipofisaria so-
metida a un control negativo por el hipotlamo.
2. La administracin intravenosa de TRH y la hipo-
glucemia insulnica estimulan la secrecin de pro-
lactina.
3. Los progestgenos estimulan la liberacin de pro-
lactina actuando sobre la hipfisis, mientras que
los estrgenos inhiben dicha produccin.
4. El estmulo de succin del pezn tras el parto pro-
voca una descarga de prolactina que fomenta la
produccin de leche y tambin una descarga de
oxitocina que contrae los folculos mamarios para
su eyeccin.
5. Actualmente se admite que la nica sustancia re-
guladora de la sntesis de prolactina es la dopami-
na, sin que exista una regulacin positiva de im-
portancia.

Centro de Estudios Aula Neptuno - GRANADA Ver.: 07/2000
www.aulaneptuno.com
- 14 -
117. Se realiza una radiografa de crneo a una mujer
obesa de 45 aos que ha sufrido un accidente de
circulacin; se observa que la silla turca est au-
mentada de tamao. En las pruebas endocrinas no
se detecta ninguna anomala y en la TAC se com-
prueba que la silla turca est vaca. Cul de los si-
guientes es el tratamiento ms apropiado para este
paciente?

1. Ciruga transesfenoidal.
2. Radioterapia.
3. Tratamiento con bromocriptina.
4. Reposicin hormonal.
5. Tranquilizar a la paciente.

118. Los glucocorticoides tienen un efecto fundamental
sobre el metabolismo de los hidratos de carbono a
nivel del hgado. Sabes cul es la principal accin
de stos en el citado rgano?

1. Inhibir la sntesis de glucosa.
2. Promover la gluconeognesis.
3. Inhibir la captacin heptica de glucgeno.
4. Aumentar la glucolisis.
5. Favorecer la glucogenolisis.

119. En una insuficiencia corticosuprarrenal primaria
(Addison), el trastorno ms probable de entre los
siguientes, es:

1. Hipocalcemia.
2. Hiponatremia.
3. Hipernatremia.
4. Hipercloremia.
5. Alcalosis hipopotasmica.

120. Un nio de 4 aos presenta un cuadro clnico con-
sistente en hipotensin arterial, prdida renal de
sodio, hiperpotasemia, hiponatremia y niveles ele-
vados de renina y aldosterona. El desarrollo sexual
es normal. Cul de los siguientes diagnsticos es el
adecuado para este caso?

1. Dficit de 17 hidroxilasa.
2. Hipoaldosteronismo secundario.
3. Sndrome de Liddle.
4. Seudohipoaldosteronismo.
5. Dficit de 18 deshidrogenasa.

121. La asociacin de hipercortisolismo, elevacin de los
niveles plasmticos de ACTH, ausencia de supre-
sin con 8 mg de dexametasona y captacin bilate-
ral en la gammagrafa con 19-yodocolesterol su-
giere en primer lugar:

1. Cushing yatrognico.
2. Cushing por adenoma suprarrenal.
3. Enf.de Cushing.
4. S.de Cushing por carcinoma suprarrenal.
5. Cushing ectpico.

122. Una mujer diagnosticada de insuficiencia supra-
rrenal primaria refiere un cuadro de fiebre eleva-
da, dolor torcico con tos y expectoracin desde
hace 2 das. En las ltimas 4 horas presenta vmi-
tos incoercibles y dolor abdominal. Cul de las si-
guientes actitudes le parece ms correcta?:

1. Aumentar la dosis de esteroides por va oral al do-
ble y aumentar la ingesta de sal.
2. Dar antiemticos y aumentar la dosis de esteroides
orales.
3. Inicar tratamiento antibitico para una neumona
adquirida en la comunidad y aumentar al doble la
dosis habitual de esteroides.
4. Dar antiemticos y si no responde iniciar hidrata-
cin intravenosa.
5. Hidratacin intravenosa y esteroides a altas dosis
por va intravenosa.

123. Entre las enfermedades asociadas con los sndro-
mes pluriglandulares inmunes no se encuentra:

1. Enf.de Wilson.
2. Vitligo.
3. Celiaqua.
4. Eritroblastopenia pura.
5. Infecciones por Giardia.

124. En un tumor carcinoide intestinal localizado, cul
de estos sntomas podramos encontrar?

1. Hemorragia digestiva.
2. Bochornos.
3. Polipnea.
4. Ulcera gstrica.
5. Hepatomegalia.

125. En relacin al bocio simple, es falsa una de las
siguientes consideraciones:

1. En la casi totalidad de los pacientes, la TSH est
por encima de los niveles normales.
2. Los pacientes suelen ser clnicamente eutiroideos.
3. La transformacin hacia bocio multinodular txi-
co es ms frecuente a mayor tiempo de evolucin.
4. El tratamiento mdico, en ausencia de transforma-
cin noduar, es la levotiroxina.
5. La ronquera por lesin del nervio recurrente debe
hacer pensar en una neoplasia.

126. En un grupo de 250 individuos, 100 tienen una
enfermedad X. Se aplica un test con una sensibili-
dad del 70% y una especificidad del 100%. El va-
lor predictivo positivo del test ser:

1. 30%.
2. 40%.
3. 80%.
4. 90%.
5. 100%.

127. El valor predictivo positivo de un test sobre una
cierta enfermedad en una comunidad, es del 11%.
El valor predictivo negativo es del 91%. Si hay
1000 individuos y 100 de ellos estn enfermos, y
hay 50 falsos negativos, cul es la sensibilidad del
test?
Centro de Estudios Aula Neptuno - GRANADA Ver.: 07/2000
www.aulaneptuno.com
- 15 -

1. 46%.
2. 72%.
3. 50%.
4. 93%.
5. 97,5%.

128. Dos comunidades A y B tienen igual tasa bruta de
mortalidad anual. La mortalidad ajustada por
edad de la comunidad B es mayor que la existente
en la comunidad A. Qu interpretacin es la co-
rrecta?

1. La edad de la poblacin es similar en ambas co-
munidades.
2. Las peores condiciones higinico-sanitarias co-
rresponden a la comunidad A.
3. La poblacin de A es ms vieja que la de B.
4. Las mejores condiciones higinico-sanitarias se
dan en la comunidad B.
5. Ambas poblaciones tienen igual nivel de asisten-
cia sanitaria.

129. En cul de los siguientes casos no es apropiado
efectuar un ensayo clnico cruzado para evaluar la
eficacia comparativa de dos tratamientos?

1. Para comparar la eficacia de dos frmacos antian-
ginosos en la mejora de la tolerancia al esfuerzo.
2. Para evaluar la eficacia de dos tratamientos
ansiolticos en pacientes con trastorno por
ansiedad generalizada.
3. Para comparar la reduccin de las cifras de coles-
terol de dos tratamientos hipolipemiantes.
4. Para comparar los resultados de dos pautas de
erradicacin del Helicobacter pylori.
5. Para comparar la eficacia de dos tratamientos en el
control de los sntomas de colon irritable.

130. Un estudio epidemiolgico en el que la asignacin
del factor de estudio corresponde a los investigado-
res y en el que no hay asignacin aleatoria de los
individuos a los distintos grupos de estudio, se de-
nomina:

1. Ensayo clnico.
2. Estudio de cohortes.
3. Serie de casos clnicos.
4. Estudio de intervencin comunitaria.
5. Estudio de casos y controles.

131. SI en el MIR te encuentras con una pregunta en la
que se te pide calcular la proporcin de riesgo atri-
buible, qu es lo que realmente quieren que res-
pondas?

1. La fraccin etiolgica.
2. El riesgo relativo.
3. El riesgo atribuible poblacional.
4. La fraccin etiolgica poblacional.
5. El nmero total de casos de la enfermedad en ex-
puestos.

132. Se sabe que el riesgo relativo de una persona HBs
Ag negativa de desarrollar carcinoma hepatocelu-
lar si est expuesta a la aflatoxina es de 1,9. Sin
embargo, dicho riesgo relativo aumenta a 12,5 en-
tre individuos HBs Ag positivos. El efecto produci-
do por el virus B entre los expuestos a aflatoxina se
denomina:

1. Confusin.
2. Interaccin.
3. Regresin.
4. Multivariacin.
5. Insatisfaccin.

133. Cul de las siguientes afirmaciones sobre los
sesgos que se pueden cometer en un estudio es fal-
sa?

1. La asignacin aleatoria previene los sesgos de se-
leccin.
2. Las tcnicas de enmascaramiento previenen los
sesgos de informacin.
3. Cuanto mayor sea el tamao de la muestra, menor
es la probabilidad de cometer un sesgo de selec-
cin.
4. Las tcnicas de anlisis multivariante no permiten
controlar los sesgos de informacin.
5. Las tcnicas de anlisis multivariante no permiten
controlar los sesgos de seleccin.

134. De los criterios que se sealan a continuacin to-
dos, excepto uno, justifican un programa de scree-
ning; indique cul:

1. Que la enfermedad sea potencialmente grave.
2. Que carezca de fase latente.
3. Que sea tratable.
4. Que sea vulnerable al tratamiento.
5. Que el test de screening sea aceptado por la pobla-
cin.

135. Sobre la circulacin pulmonar, cul es falsa?

1. Normalmente el sistema nervioso autnomo tiene
muy poco efecto sobre la microcirculacin pul-
monar.
2. Cuando pequeas arterias pulmonares estn
ocluidas por mbolos se produce un reflejo vaso-
constrictor generalizado en el rbol vascular pul-
monar.
3. La estimulacin simptica tiene especial impor-
tancia, sobre todo, sobre los grandes vasos pulmo-
nares de capacitancia (venas).
4. En la persona en posicin erecta, la presin arterial
pulmonar es mucho mayor en los vrtices que en
las bases, lo que explica que el flujo sea menor.
5. En los vrtices el flujo sanguneo es intermitente,
lo que significa que slo se verifica en sstole.

136. Entre los cambios del flujo pulmonar producidos
normalmente con el ejercicio fsico, no se encuen-
tra:

1. Aumento de la presin arterial pulmonar en 5
mm.Hg.como trmino medio.
Centro de Estudios Aula Neptuno - GRANADA Ver.: 07/2000
www.aulaneptuno.com
- 16 -
2. Establecimiento de una relacin ventilacin-
perfusin prxima a la ideal en los vrtices.
3. Aumenta la velocidad de flujo.
4. Aumento del nmero de capilares abiertos.
5. Reduccin de las resistencias vasculares pulmona-
res.

137. Sobre la distribucin de los lquidos en el cuerpo
humano-a, es falso que:

1. En el compartimento transcelular hay unos 10 li-
tros de lquidos varios.
2. El plasma representa unos tres litros de lquido.
3. En el lquido intersticial hay unos 11 litros.
4. Todo el compartimento extracelular alberga unos
14 litros.
5. EL compartimento intracelular tiene, aproxima-
damente, el doble de lquido que el extracelular
(unos 28 litros).

138. EL Anhdrido carbnico (CO2) que circula en
sangre unido a hemoglobina forma la llamada:

1. Carbonihemoglobina.
2. Carboxihemoglobina.
3. Carbenicilina.
4. Carbaminohemoglobina.
5. Anhidrohemoglobina.

139. Los ncleos gracilis y cuneiforme son importantes
en:

1. La transmisin de la sensibilidad somtica.
2. La activacin motora por el haz rubroespinal.
3. La funcin de los ganglios basales.
4. Cualquier funcin cerebelosa, ya que son los dos
principales ncleos grises, junto al dentado, del ce-
rebelo.
5. La transmisin de la sensibilidad termoalgsica.

140. En cul de las siguientes enfermedades no existe
acantolisis?

1. Pnfigo vulgar.
2. Pnfigo benigno familiar.
3. Enfermedad de Darier.
4. Penfigoide ampolloso.
5. Dermatosis acantoltica transitoria.

141. Respecto a la inmunofluorescencia directa en piel,
cul es falso?

1. En el pnfigo vulgar existe depsito de IgG a nivel
intercelular epidrmico.
2. En el penfigoide ampolloso existe depsito de IgG
y C3 lineal a nivel de la unin dermo-epidrmica.
3. En la dermatitis herpetiforme existen depsitos de
IgA en papilas drmicas.
4. En el herpes gestationis existe depsito de IgG y
C3 en unin dermo-epidrmica.
5. En la epidermolisis ampollosa distrfica hay de-
psito de IgG, IgA y C3 a nivel de la unin dermo-
epidrmica.

142. El nivel III de la clasificacin de CLARK en el
melanoma maligno corresponde a una de las si-
guientes proposiciones. Indquela:

1. Invasin de la epidermis.
2. Invasin de la dermis papilar parcialmente.
3. Invasin de la dermis papilar totalmente.
4. Invasin de la dermis reticular parcialmente.
5. Invasin de la hipodermis.

143. La liquenificacin flexural es caracterstica habi-
tual en:

1. Dermatitis de contacto.
2. Eczema discoide.
3. Eczema seborreico.
4. Eczema microbiano.
5. Dermatitis atpica.

144. Un hombre de 70 aos consulta por ampollas
dolorosas que se rompen fcilmente en muchas
partes del cuerpo: cara, tronco y cavidad bucal en-
tre ellas. La presin mecnica sobre la piel provoca
la separacin de las capas de la epidermis. No tiene
nign otro problema mdico importante ni tom
ningn frmaco antes de comenzar la erupcin.
Slo es falsa una de las siguientes acerca de este
cuadro:

1. La biopsia de las lesiones mostrar una banda li-
neal de C3 en la zona de la membrana basal.
2. Si no se trata, la enfermedad tiene una mortalidad
superior al 50%.
3. Tras la confirmacin diagnstica, el enfermo debe
comenzar tratamiento con glucocorticoides.
4. La cavidad de las ampollas contiene clulas epi-
drmicas desprendidas.
5. Puede verse afectacin del recto y el esfago.

145. Una de estas sustancias no suele reabsorberse de
forma activa en el tbulo proximal:

1. Cloro.
2. Sodio.
3. Fosfatos.
4. Aminocidos.
5. Glucosa.

146. La capacidad de concentracin urinaria del rin
puede medirse por:

1. Presencia de cilindros en el sedimento.
2. Prueba de la sobrecarga acuosa.
3. Aclaramiento de inulina.
4. Prueba de la deshidratacin.
5. Concentracin de electrolitos en orina.

147. Segn el algoritmo diagnstico a aplicar ante un
paciente con hematuria, si no va acompaada de
leucocituria, proteinuria, cilindros hemticos,
hemates dismrficos ni hipercalciuria, cul ser
la primera prueba diagnstica a aplicar?

1. Biopsia renal.
Centro de Estudios Aula Neptuno - GRANADA Ver.: 07/2000
www.aulaneptuno.com
- 17 -
2. Cultivo de orina.
3. TAC.
4. Urografa intravenosa.
5. Ecografa.

148. En un paciente con sndrome nefrtico se produce
un aumento brusco de la proteinuria, deterioro de
la funcin renal y aumento del tamao de las silue-
tas renales. Dgto.ms probable:

1. Embolismo de la arteria renal.
2. Pielonefritis aguda.
3. Trombosis venosa renal.
4. Necrosis tubular aguda.
5. Necrosis papilar.

149. Un estudiante de 28 aos presenta una creatinina
elevada (3,5 mg/dl). Refiere polidipsia, poliuria y
nicturia. En las pruebas complementarias destaca
una acidosis metablica hiperclormica y leucoci-
turia con urocultivos negativos. Cul de las que se
mencionan aparecer ms precozmente en este
cuadro?

1. Dificultad para concentrar la orina.
2. Hematuria.
3. Hiperlipoproteinemia.
4. Proteinuria de rango nefrtico.
5. Anticuerpos anti membrana basal glomerular.

150. La Nefropata de los Balcanes parece estar relacio-
nada con uno de los siguientes agentes infecciosos.
Cul?

1. Esquistosoma.
2. Coronavirus.
3. Citomegalovirus.
4. Ureaplasma.
5. Rickettsias.

151. Sobre las caractersticas generales de las nefropat-
as intersticiales, indica la respuesta falsa:

1. Las formas agudas pueden acompaarse de insu-
ficiencia renal aguda con hiperpotasemia y acido-
sis metablica, que suele requerir dilisis.
2. En las formas crnicas se alteran precozmente las
pruebas tubulares con cifras de creatinina y urea
normales o moderadamente elevadas.
3. La alteracin ms precoz y caracterstica en las ne-
fropatas intersticiales es la prdida de la capacidad
de concentracin urinaria.
4. En estas enfermedades se pierde la capacidad de
reabsorber sodio en los tramos finales de la nefro-
na, ocasionndose hipovolemia.
5. La capacidad de excretar potasio est conservada
hasta fases avanzadas de reduccin del filtrado
glomerular.

152. Paciente de 6 aos que presenta poliuria, polidip-
sia, astenia, anorexia y algn episodio convulsivo.
No edemas. TA: 110/65. Analtica: Potasio:
2.2mEq/L.; sodio 133 mEq/L.; Cloro 98 mEq/L.;
ARP 6.3 ng/mL/hora.; aldosterona 19.2 ng/dL.;
pH: 7.503.; bicarbonato 37.4 mEq/L. Hay una ori-
na hipotnica con cloro urinario>20mEq/l y un
aumento de la concentracin urinaria de PGE2.
Cul de los siguientes tratamientos no indicaras
en este nio?

1. Captopril.
2. Indometacina.
3. Cloruro potsico.
4. Hidroclorotiacida.
5. Espironolactona.

153. Paciente varn de 68 aos diagnosticado de tuber-
culosis pulmonar. Inicia tratamiento con isoniaci-
da, rifampicina y etambutol. Al mes abandona la
medicacin por gastroenteritis, reanudndola a las
dos semanas. A los 12 das de reiniciado el trata-
miento presenta un cuadro de fiebre de 38C,
escalofros, mialgias y artralgias generalizadas,
nuseas y vmitos, sensacin de malestar general y
oliguria, por lo que fue hospitalizado. En la
analtica destacan: Datos de insuficiencia renal,
aumento de IgE, complemento normal. En el
sedimento, proteinuria de 1,2 g/24 horas,
microhematuria, leucocituria y eosinofiluria. El
diagnstico ms probable es:
1. Nefritis tubulointersticial aguda por rifampicina.
2. Afectacin tuberculosa pielocalicial.
3. Sndrome Urmico Hemoltico.
4. Pielonefritis aguda, sin que tenga nada que ver con
la tuberculosis.
5. Necrosis tubular por etambutol.

154. El tratamiento de eleccin de la arreflexia refleja
del detrusor es:

1. Autosondaje.
2. Alfa bloqueantes.
3. Relajantes musculares.
4. Diacepam intravenoso.
5. Parasimpaticomimticos.

155. Paciente con acidosis metablica, hipercloremia,
hipopotasemia, pH urinario de 7,2 tras adminis-
trar cloruro amnico, y ausencia de elevacin de la
PCO2 urinaria tras forzar diuresis alcalina. Tra-
tamiento:

1. Bicarbonato y suplementos de calcio.
2. Bicarbonato y cloruro potsico.
3. Furosemida y potasio.
4. Slo bicarbonato.
5. Bicarbonato, potasio y furosemida.

156. Cul es actualmente la primera exploracin dia-
gnstica que debe realizarse en la evaluacin de
una masa renal?

1. Arteriografa renal.
2. TC abdominal con contraste.
3. Urografa intravenosa.
4. Puncin aspiracin con aguja fina.
5. Ecografa renal.

Centro de Estudios Aula Neptuno - GRANADA Ver.: 07/2000
www.aulaneptuno.com
- 18 -
157. En un varn de 50 aos de edad asintomtico se
realiza un estudio de hipertensin arterial. El se-
dimento urinario es normal. El nico hallazgo ob-
tenido en la urografa iv. es una masa nica radio-
transparente de 2,7 cm de dimetro en el polo
superior del rin derecho. La actitud diagnstica
ms coherente sera:

1. Repetir la urografa al ao ya que por sus caracte-
rsticas radiolgicas la masa con seguridad es qus-
tica y benigna.
2. Citologa de orina.
3. RMN.
4. TC con contraste.
5. Ecografa renal y eventual aspiracin con aguja de
la masa.

158. Paciente de 66 aos acude a su consulta por cua-
dro de hematuria autolimitada. Se realizo eco sin
signos de hiperecogenicidad ni sombras anecoides.
Se realizo urografia que mostro defecto de la re-
pleccion a nivel de pelvis menor del rion derecho.
Cual de los siguientes es su diagnostico de sospe-
cha:

1. Carcinoma de celulas transicionales.
2. Litiasis de cristales de hidroxiapatita.
3. Litiasis por cristales de cistina.
4. Litiasis por oxalato calcico.
5. Angiodisplasia tubular.

159. Cul de los siguientes frmacos parece beneficio-
so porque acta sobre el mecanismo patognico
principal no inmune de la hialinosis focal y seg-
mentaria (que parece ser la hiperfiltracin glome-
rular)?

1. Ciclofosfamida.
2. Captopril.
3. Azatioprina.
4. Lovastatina.
5. Dopamina.

160. Uno de los que siguen no implica mal pronstico
en un paciente con nefropata membranosa:

1. Fibrosis intersticial y atrofia tubular en la biopsia.
2. Sexo masculino.
3. Hiperlipidemia intensa.
4. Menor edad en el momento del diagnstico.
5. Proteinuria superior a 10 gramos/da.

161. Un nio que presenta por ecografa masas renales
en ambos riones que, segn se demuestra por la
TC, tienen grasa en el interior, seguramente pa-
dezca:

1. Un tumor de Wilms.
2. Una tuberculosis renal.
3. Una poliquistosis renal recesiva.
4. Una neoplasia endocrina mltiple.
5. Una facomatosis.

162. En cul de las siguientes enfermedades vasculares
del rin es ms probable la observacin de alcalo-
sis metablica?

1. Tromboembolismo renal.
2. Nefropata ateroemblica.
3. Estenosis de la arteria renal.
4. Sndrome urmico hemoltico.
5. Drepanocitosis.

163. Cul de las siguientes complicaciones no se rela-
ciona con el colesteatoma?

1. Meningitis.
2. Tromboflebitis del seno lateral.
3. Neuronitis vestibular.
4. Parlisis facial.
5. Fstula laberntica.

164. Respecto a la fstula laberntica, no es cierto que:

1. Se presenta en el 5% de las otorreas crnicas.
2. El sntoma tpico es el vrtigo.
3. La mayora son de origen colesteatomatoso.
4. Se acompaa de hipoacusia mixta.
5. Es caracterstico el nistagmo vertical.

165. En las curvas audiomtricas no es cierto que:

1. La va sea y la va area estn superpuestas en
una hipoacusia de transmisin.
2. El umbral de audicin por va sea representa la
reserva coclear y la funcionalidad de sus conexio-
nes centrales.
3. La cada audiomtrica en la frecuencia de los
4.000 ciclos/segundo es tpica del trauma acstico.
4. La cua de Carhart es tpica de la otoesclerosis.
5. La prdida auditiva entre 30 y 40 dB representa
una hipoacusia de pequeo grado.

166. En la epiglotis aguda NO hay:

1. Fiebre.
2. Disfagia.
3. Hemoptisis.
4. Estridor inspiratorio.
5. Voz gangosa.

167. La manifestacin inicial ms frecuente del cncer
de laringe en Espaa es:

1. Disnea.
2. Disfagia.
3. Parestesias.
4. Otalgia.
5. Disfona.

168. Al revisar la analitica prenatal de su paciente des-
cubre usted que es Rh negativo y Du negativo, con
una exploracion negativa para anticuerpos. La
administracion de gammaglobulina anti-D negati-
va esta indicada en cada una de las siguientes cir-
cunstancias, excepto:

1. Despues de una amniocentesis
Centro de Estudios Aula Neptuno - GRANADA Ver.: 07/2000
www.aulaneptuno.com
- 19 -
2. Despues de un procedimiento de version extena
3. En la semana 28 de gestacion
4. Durante el parto
5. 40 horas despues del parto

169. En el transcurso de la ecografia de una mujer
primigravida de 25 aos en la semana 19 de gesta-
cion, con un nivel alto de alfa-fetoproteina en suero
materno, se obtiene una imagen de onfalocele fetal.
Por lo demas, la ecografia de alta definicion (orien-
tada) es normal. Al comentar con la paciente la
atencion clinica inmediata, usted le recomienda to-
dos los pasos listados a continuacion, excepto:

1. Amniocentesis para cariotipo fetal
2. Consultar con un cirujano pediatra
3. Interrumpir la gestacion
4. Repetir la ecografia al cabo de 4 semanas
5. Cordocentesis para cariotipo fetal

170. Una nia de 3 aos de edad presenta un desarrollo
mamario correspondientea la fase II de Tanner. No
hay vello pubico ni historia de menstruacion. La
salud de la paciente es excelente. A cual de los si-
guientes fenomenos se asocia frecuentemente la te-
larquia prematura?

1. Formacion de quistes ovaricos
2. Pubertad completa precoz
3. Hamartoma de la glandula pituitaria
4. Prolactinoma
5. Estrogenos maternos

171. Con respecto a la profilaxis de la enfermedad
hemoltica perinatal seale la respuesta INCO-
RRECTA:

1. La Ig antiD no se administra a la mujer Rh+.
2. La Ig antiD no se administra a la paciente ya sen-
sibilizada.
3. La Ig antiD se administra incluso despus de abor-
tos ectpicos.
4. La Ig slo es necesario aplicarla en una ocasin y
sirve para todas las gestaciones posteriores.
5. La Ig slo se administra a pacientes Rh- que hayan
tenido un feto Rh+.

172. En un estudio comparativo entre dos frmacos
para tratamiento por va sistmica del acn, existe
una diferencia a favor de uno de ellos que no es
significativa (p<0,07). Qu decisin debe tomarse
ante este resultado?

1. Aunque no sea estadsticamente significativa, est
claro que un frmaco es mejor que otro, por lo que
publicaremos estos resultados.
2. Hay que concluir el estudio, declarando definiti-
vamente cierta Ho.
3. Parece, segn los resultados, que ambos frmacos
tienen una eficacia similar.
4. Debo aumentar el nmero de sujetos estudiados.
5. Hago trampas y falseo los resultados.

173. Para comparar dos variables no normales se usa:

1. T de Student.
2. Test de Kruskal-Wallis.
3. Test de Wilcoxon.
4. Anlisis de la varianza.
5. Test exacto de Fisher.

174. Un estudio consistente en definir cuntos microor-
ganismos de la flora colnica de un individuo se co-
lorean con la tincin de Gram, de entre las 103 co-
lonias que hemos obtenido en una muestra, sigue
una distribucin:

1. Normal.
2. Binomial.
3. Poisson.
4. Normal tipificada.
5. Cohortes microscpicas

175. Cul es falsa en referencia a la prueba de la chi
cuadrado?

1. El clculo de la chi cuadrado experimental se ob-
tiene mediante la sumatoria de los cocientes: |oi -
ei|2 / ei.
2. Los grados de libertad se calculan multiplicando el
nmero de filas menos uno por el nmero de co-
lumnas menos uno.
3. Un valor alto de este estadstico significa que hay
mucha diferencia entre los valores observados y
los valores esperados, indicando que existe una re-
lacin entre las variables comparadas.
4. Existe el acuerdo de que la prueba no es aplicable
si alguna frecuencia esperada es inferior a 2.
5. Si el nmero de filas y/o columnas es superior a 4,
debe aplicarse la correccin de Yates.

176. El anlisis de la varianza no paramtrico para
muestras independientes se denomina:

1. Test de Tukey.
2. Test de Scheff.
3. Test de Newman-Keuls.
4. Test de Friedman.
5. Test de Kruskal Wallis.

177. Para comparar la mejora de la cefalea tensional
en un grupo de pacientes con dicho diagnstico se
han realizado tres periodos sucesivos de tratamien-
to en ellos con sendos frmacos. Cul es la prueba
estadstica ms adecuada para efectuar dicha
comparacin?

1. Anlisis de la varianza simple.
2. Test de la Q de Cochran.
3. Test de Mc Nemar.
4. Anlisis de la varianza de medidas repetidas.
5. T de Student para datos apareados.

178. En una muestra de 1000 sujetos de una poblacin,
250 de ellos son fumadores. El error estndar de la
proporcin es, aproximadamente, 0,01. Qu
afirmacin es cierta?

Centro de Estudios Aula Neptuno - GRANADA Ver.: 07/2000
www.aulaneptuno.com
- 20 -
1. El porcentaje de fumadores en la poblacin de la
que procede la muestra es del 25%.
2. Existe un 95% de confianza de que la verdadera
proporcin de fumadores se site entre el 24 y el
26%.
3. Existe un 95% de confianza de que la verdadera
proporcin se site entre el 23 y el 27%.
4. El 95% de los grupos de tamao 1000 que se ob-
tengan tendrn una proporcin de fumadores que
se situar entre el 24 y el 26%.
5. Hay un 95% de probabilidades de que en la po-
blacin de referencia el porcentaje de fumadores
sea del 25%.

179. El test exacto de Fisher es una prueba para:

1. Contrastar que una variable sigue una distribucin
normal.
2. Contrastar la igualdad de dos proporciones cuando
el tamao muestral es pequeo.
3. Contrastar la igualdad de dos proporciones cuando
el tamao muestral es grande.
4. Contrastar la igualdad de dos medias cuando el
tamao muestral es pequeo.
5. Contrastar la igualdad de dos medias cuando el
tamao muestral es grande.

180. Si tenemos una variable con distribucin N (3, 1),
cul es la probabilidad aproximada del intervalo
(1, 5)?

1. 0,95.
2. 0,99.
3. 0,68.
4. 0,64.
5. 0,84.

181. Todos los frmacos que se citan, excepto uno, pue-
den ser tiles para evitar la oclusin brusca por
espasmo o la formacin de un trombo en una
arteria coronaria dilatada mediante ACTP:

1. Nitratos.
2. Aspirina.
3. Heparina.
4. Antagonistas del calcio.
5. Beta bloqueantes.

182. En cul de las siguientes situaciones no se encon-
trar nunca una onda a gigante en el pulso venoso
yugular?

1. Estenosis pulmonar.
2. Bloqueo auriculoventricular total.
3. Fibrilacin auricular.
4. Taquicardia ventricular.
5. Ritmos de origen nodal.

183. De los siguientes parmetros que valoran la grave-
dad de una estenosis mitral, cul es el ms fiable y
objetivo a la hora de establecer la posibilidad de
indicacin quirrgica?

1. El rea valvular.
2. La distancia entre el segundo ruido y el chasquido.
3. La presin arterial pulmonar.
4. La intensidad del soplo.
5. El dimetro de la aurcula izquierda.

184. En el diagnstico diferencial entre miocardiopata
restrictiva y pericarditis constrictiva, uno de los si-
guientes datos es ms caracterstico de la segunda
que de la primera:

1. Una presin sistlica de la arteria pulmonar supe-
rior a 50 mm.Hg.
2. La presin en la aurcula derecha es igual a la pre-
sin de enclavamiento pulmonar.
3. La fraccin de eyeccin est disminuida.
4. El volumen telediastlico del ventrculo izquierdo
est ligeramente aumentado.
5. No se observa engrosamiento pericrdico.

185. Todas las entidades clinicopatolgicas siguientes
poseen relacin etiolgica con los aneurismas de
aorta menos una. Seale cul es:

1. Necrosis qustica de la media.
2. Enfermedad de BUERGER.
3. Sfilis.
4. Sindrome de MARFAN.
5. Arterioesclerosis.

186. Cul de los siguientes vasodilatadores es arterial
puro:

1. Captopril.
2. Nitroprusiato.
3. Prazosin.
4. Dinitrato de isosorbida.
5. Fentolamina.

187. Cul de las siguientes NO es causa de Taquicar-
dia Ventricular tipo "torsade de pointes" ? :

1. Sd. de Jervell y Large-Nielsen.
2. Hiperpotasemia.
3. Quinidina.
4. Insecticidas.
5. Antidepresivos Tricclicos.

188. Cul de los siguientes fenmenos auscultatorios
en un paciente con Estenosis Mitral indica que est
en ritmo sinusal?

1. Retumbo diastlico.
2. Chasquido de apertura.
3. Primer ruido fuerte.
4. Desdoblamiento patolgico del segundo ruido.
5. Refuerzo presistlico.

189. En un paciente con doble lesin mitral no calcifi-
cada sintomtica y en grado funcional III-IV de la
NYHA a pesar del tratamiento mdico, la opcin
ms aceptable es:

1. Valvuloplastia mitral percutnea.
2. Comisurotoma cerrada.
Centro de Estudios Aula Neptuno - GRANADA Ver.: 07/2000
www.aulaneptuno.com
- 21 -
3. Comisurotoma a corazn abierto.
4. Comisrotoma y anuloplastia.
5. Sustitucin valvular.

190. Cul es falsa respecto al ECG de la pericarditis
aguda y su diferenciacin con procesos isqumicos
coronarios?

1. En la pericarditis aguda existen elevaciones del
segmento ST en algunas derivaciones pero sin que
existan descensos recprocos en otras.
2. En las pericarditis agudas no coexisten habitual-
mente la elevacin del segmento ST con la inver-
sin de las ondas T.
3. En las pericarditis agudas no se observan cambios
en el QRS, salvo disminucin de voltaje en pa-
cientes con grandes derrames pericrdicos.
4. En el IAM la elevacin del segmento ST suele
normalizarse ms rpido que en las pericarditis.
5. En las pericarditis agudas los cambios ECG suelen
observarse en varias derivaciones estndar de los
miembros y desde V2 hasta V6.

191. Una mujer joven se presenta con hemorragia
despus de una endodoncia. Se encuentra que tie-
ne un tiempo de sangra mayor de 20 minutos, con
tiempos de protrombina y parcial de tromboplas-
tina normales. Hay antecedentes familiares de
hemorragia y la valoracin de laboratorio revela
una enfermedad de von Willebrand tipo IIa. La
actividad coagulante del factor VIII es del 54%, el
Ag.del vW es del 48% y el cofactor ristocetina es
del 13%. El patrn anormal de multmeros vWF
del plasma de la paciente en la electroforesis con
gel de agarosa se debe a:

1. Liberacin defectuosa de vWF a partir de clulas
endoteliales.
2. Unin inapropiada de vWF a las plaquetas.
3. Sntesis reducida de vWF por clulas endoteliales.
4. Incapacidad para ensamblar multmeros de alto
peso molecular o catabolismo prematuro de vWF.
5. Alteracin en le receptor plaquetario para vWF.

192. Cul de los siguientes datos descarta mejor el
dgto.de una hemofilia A clsica?

1. La ausencia de antecedentes familiares.
2. La ausencia de fenmenos hemorrgicos.
3. La ausencia de hemorragias tras la ciruga.
4. Un TTPA normal.
5. Un tiempo de Quick normal.

193. En un paciente diagnosticado de Leucemia-
Linfoma de cl. T del adulto esperaras encontrar
todo lo siguiente, excepto :

1. Linfadenopatas perifricas.
2. Eritrodermia.
3. Fenotipo celular con abundantes Ig de superficie.
4. Poliuria y polidipsia.
5. Infeccin por HTLV-I.

194. Cul de las siguientes trombocitopenias suele
cursar con vida media plaquetaria acortada y au-
mento de megacariocitos en mdula sea?

1. Leucemia aguda.
2. Enf.de Werlhof.
3. Mielofibrosis idioptica.
4. S.de Fanconi.
5. S.de Wiskott-Aldrich.

195. La causa ms frec. de atelectasia redonda (Sd. de
Blesovsky) es :

1. Ca broncognico.
2. TBC pulmonar.
3. Exposicin al asbesto.
4. Silicosis.
5. Beriliosis.

196. La principal caracterstica patognica de la
Hb.Barts reside en su capacidad para:

1. Desplazar la curva de saturacin de Hb.a la dere-
cha.
2. Deformar la membrana del hemate.
3. Precipitar, formando grumos.
4. Unirse a hierro frrico.
5. Unirse al O2, con una gran afinidad.

197. Existe una anemia hemoltica de causa intrnseca
en la que se observan de forma simultnea hemli-
sis, eritropoyesis ineficaz y hemoglobinizacin de-
fectuosa. Nos estamos refiriendo a:

1. El dficit de glucosa 6 fosfato deshidrogenasa.
2. La esferocitosis hereditaria.
3. La drepanocitosis.
4. La beta talasemia.
5. La hemoglobinuria paroxstica nocturna.

198. En un paciente con aplasia medular grave en el
que no puede realizarse trasplante de mdula sea
por la edad, el tratamiento que debe ensayarse es:

1. Ciclofosfamida.
2. Danazol y carbonato de litio.
3. Adriamicina.
4. Desoxicoformicina.
5. Globulinas antilinfoctica y antitimoctica.

199. Varn de 5 aos de edad que acude a urgencias del
hospital porque desde hace unas horas padece fie-
bre de 39,5C y dolor abdominal agudo. Simult-
neamente ha presentado orinas muy oscuras. Tres
das antes fue visto por su pediatra por un cuadro
de fiebre de mediana intensidad y dolor en la gar-
ganta de 24 horas de evolucin, siendo diagnosti-
cado de amigdalitis e iniciado tratamiento con una
sulfamida, que sigue en la actualidad. Exploracin:
Sensacin de enfermedad grave. Palidez intensa de
piel y marcada ictericia de mucosas. Soplo sistlico
polifocal II/VI. Abdomen doloroso a la palpacin
sin visceromegalias. Analtica: Ht: 2.347.000/mm3;
Hb: 6,7 g/dl; Hto: 21%VSG 47/73; Reticulocitos:
257.000/mm3; Cuerpos de Heinz; Elevacin de
Centro de Estudios Aula Neptuno - GRANADA Ver.: 07/2000
www.aulaneptuno.com
- 22 -
BRR total e indirecta y LDH. Hemoglobinuria in-
tensa en orina. Test de Coombs(-); Ham y sucrosa
(-); ROE normal. La causa ms probable de este
trastorno es:

1. Talasemia maior.
2. Anemia por anticuerpos calientes.
3. HPN.
4. Dficit de G-6-PD.
5. Hiperesplenismo.

200. Sobre los aspectos generales del diagnstico de las
anemias hemolticas, seala la respuesta falsa:

1. En el hemograma suele comprobarse un descenso
de la concentracin de hemoglobina, con un ligero
aumento del volumen corpuscular medio.
2. El marcado de los hemates del paciente con cro-
mo-51 es el mtodo ms directo y preciso para de-
terminar la supervivencia media eritrocitaria.
3. En ausencia de hemoglobinuria, la coloracin de
la orina es normal, aunque exista ictericia.
4. Las cifras normales de reticulocitos oscilan entre
los 25.000 y 75.000/mm3.
5. Las cifras sricas de haptoglobina son prctica-
mente nulas tanto en las hemlisis como en las
hemorragias intensas.

201. Uno no es predictor de transformacin blstica en
la LMC:

1. Aumento del nmero de hemates y plaquetas.
2. Aumento del ndice de fosfatasa alcalina leucoci-
taria.
3. Aumento del tamao de hgado y bazo.
4. Lesiones seas.
5. Aumento del nmero de basfilos.

202. Una de las siguientes afirmaciones no es cierta en
la policitemia vera:

1. Las clulas eritroides de los pacientes con polici-
temia vera responden a la eritropoyetina.
2. Se ha descrito una alta incidencia en personas ex-
puestas a radiaciones ionizantes.
3. La determinacin de la eritropoyetina srica es el
principal dato para diferenciarla de las formas se-
cundarias.
4. Es poco frecuente en la raza negra.
5. Existe un aumento de progenitores mieloides y
eritroides en mdula sea.

203. Los padres de un nio consultan porque ste bebe
y orina ms de lo normal. Se realiza la prueba de
la deshidratacin y se observa que la concentra-
cin urinaria aumenta considerablemente tras
administrar ADH. En la inspeccin nos parece
apreciar una cierta prominencia ocular. Cul de
las siguientes te parece una actitud diagnstica ms
apropiada en este momento?

1. Estudio de anticuerpos antitiroideos y prueba de
estimulacin con TRH.
2. Anlisis del sedimento de orina.
3. Determinacin del pH plasmtico y clculo del
anin GAP.
4. Radiografa de crneo.
5. Biopsia cutnea.

204. Una mujer de 89 aos acude al hospital con tos
productiva, falta de aliento y fiebre. Ha sufrido dos
episodios de neumona neumoccica en los 6 lti-
mos meses y recientemente un dolor en la regin
lumbar. En la exploracin fsica est plida, con
una temperatura de 38,3C. No presenta signos de
deplecin de volumen. En la Rx.de trax se obser-
va un infiltrado en el lbulo inferior izquierdo. En
la ecografa renal no hay signos de obstruccin. En
los anlisis destacan: sodio 144; potasio 5,1; cloro
117; bicarbonato 17 urea 42; creatinina 2,3; calcio
11,4, Hb 9 y proteinuria. El dgto.ms probable es:

1. Amiloidosis.
2. Mieloma mltiple.
3. Glomerulonefritis mesangiocapilar.
4. IRC con neumona sobreaadida.
5. Adenocarcinoma renal.

205. Un paciente de 58 aos es estudiado por un cuadro
inespecfico prolongado con fiebre, sudoracin
nocturna y prdida de peso. Dice tambin padecer
sensacin de distensin abdominal, comprobndo-
se exploratoriamente esplenomegalia importante.
Existe una gran leucocitosis (140.000/mm3) con un
2% de blastos y una actividad de fosfatasa alcalina
leucocitaria casi nula. El estudio de mdula sea
muestra una translocacin recproca (9,22). Qu
es falso acerca del tratamiento de dicha enferma?

1. La hidroxiurea es en la actualidad el frmaco de
eleccin para el tratamiento convencional de la fa-
se crnica de la LMC.
2. Con el fin de prevenir la nefropata hiperuricmica
la quimioterapia debe complementarse con la ad-
ministracin de alopurinol, una ingesta lquida
abundante y alcalinizacin de la orina.
3. El resultado del trasplante de mdula sea es con-
siderablemente peor en los menores de 40 aos.
4. Los principales problemas del busulfn son su po-
tencial complicacin con aplasia medular y fibro-
sis pulmonar.
5. La hidroxiurea tiene escaso efecto mielotxico pe-
ro parece que puede ser teratgeno administrado
durante el primer trimestre del embarazo.

206. Una proposicin es incorrecta en relacin a las
anemias sideroblsticas:

1. La presencia de sideroblastos anulares en indivi-
duos alcohlicos suele desaparecer varios das
despus de suprimir la ingesta de alcohol.
2. Despus de tratar el mieloma mltiple con
quimioterpicos alquilantes puede aparecer una
forma intratable de anemia sideroblstica.
3. La anemia sideroblstica hereditaria responde al
tratamiento con altas dosis de vitamina B6.
4. La anemia refractaria sideroblstica es propia de
personas de edad avanzada y suele acompaarse
Centro de Estudios Aula Neptuno - GRANADA Ver.: 07/2000
www.aulaneptuno.com
- 23 -
de leuco y trombocitosis.
5. Las anemias sideroblsticas, en general, se caracte-
rizan por eritropoyesis ineficaz y un incremento
marcado en el hierro srico y en la saturacin de la
transferrina.

207. Qu mecanismo de anemia megaloblstica por
frmacos est mal descrito?

1. El metotrexate es un potente inhibidor de la dihi-
drofolato reductasa; tambin lo son, aunque ms
dbiles, pentamidina, trimetroprim, triamterene y
pirimetamina.
2. La zidovudina (AZT) provoca una disminucin
de la absorcin intestinal de la vitamina B12.
3. La inhalacin de xido nitroso provoca la destruc-
cin de la cobalamina endgena.
4. Los anlogos de la purina (6-tioguanina, azatiopri-
na, 6-mercaptopurina) y de la pirimidina (5-
fluorouracilo, arabinsido-C) interrumpen la snte-
sis de ADN.
5. La difenilhidantona parece interferir la absorcin
intestinal de folatos.

208. La aparicin de una intensa hemlisis en medio
cido asociada a multinuclearidad de los precurso-
res de la serie roja es indicativa del diagnstico de:

1. Esferocitosis hereditaria.
2. Anemia por anticuerpos calientes.
3. Hemoglobinuria paroxstica nocturna.
4. Anemia diseritropoytica.
5. Rasgo talasmico alfa.

209. En un enfermo anmico la presencia de abundan-
tes hemates con espculas regulares ("en rueda
dentada") en la extensin de sangre perifrica,
hace especialmente recomendable la determina-
cin de:

1. Transaminasas, bilirrubina y fosfatasa alcalina.
2. Aclaramiento renal de la creatinina.
3. Concentracin de gamma GT en suero.
4. Niveles de apolipoprotenas en sangre.
5. Niveles de porfirinas en orina.

210. En un nio con clnica de hemorragias cutneo
mucosas e historia familiar positiva, para poder
emitir el diagnstico de Sndrome de Bernard Sou-
lier, deben cumplirse todos los siguientes requisitos,
excepto:

1. El recuento plaquetario debe estar por debajo de lo
normal.
2. El tiempo de sangra estar prolongado.
3. La agregacin en presencia de ristocetina ser de-
fectuosa.
4. El trastorno no se corrige con la reposicin de
plasma fresco.
5. EL tiempo de tromboplastina parcial estar pro-
longado.

211. El diagnstico de eleccin de las lesiones agudas de
la mucosa gstrica (LAMG) es:

1. Clnico.
2. Radiografa simple.
3. Radiologa con contraste.
4. TAC.
5. Endoscpico.

212. Una asociacin de las mencionadas, con relacin a
las gastritis, no es cierta:

1. Crnica erosiva - Ac.anti-clulas parietales.
2. LAMG por shock hipovolmico - Lesiones
necrtico-hemorrgicas.
3. Crnica tipo B - Localizacin antral.
4. Ulcera aguda - Afectacin de la muscularis muco-
sae.
5. Crnica tipo A - Riesgo de cncer gstrico.

213. En un paciente con insuficiencia pancretica exo-
crina estn alteradas las siguientes pruebas excep-
to:

1. Bentiromida.
2. Pancreocimina.
3. Test de Lundh.
4. Trioleina marcada.
5. Lactulosa marcada.

214. La mejor prueba para valorar la extensin de la
enfermedad inflamatoria intestinal (EII), en el
transcurso de un brote grave de la misma es:

1. Gammagrafa.
2. Rectosigmoidoscopia.
3. Colonoscopia.
4. Trnsito baritado por enteroclisis.
5. Enema opaco.

215. De las siguientes afirmaciones sobre el sndrome
carcinoide, cul es falsa?

1. Puede presentarse como hipotensin paroxstica,
diarrea y rubefaccin.
2. Puede desarrollarse con la ingestin de alcohol.
3. La deteccin de niveles elevados de 5-HIAA en
orina tiene una sensibilidad del 100%.
4. La diarrea puede ser controlada con ciproheptadi-
na.
5. La rubefaccin es debida a actuacin del sistema
de las cininas.

216. La estrangulacin es la forma de obstruccin intes-
tinal gravada con una mayor mortalidad. A este
respecto, Cul de los siguientes signos despierta
mayor sospecha de estrangulacin en un enfermo
con un proceso oclusivo?

1. Emisin de vmitos fecaloideos.
2. Mayor intensidad del dolor.
3. Ausencia de gas radiolgicamente detectable en
los segmentos distales a la oclusin.
4. La auscultacin revela borborigmos agudos que
coinciden con el dolor clico.
5. En la radiografa se observa una distensin con
Centro de Estudios Aula Neptuno - GRANADA Ver.: 07/2000
www.aulaneptuno.com
- 24 -
gas limitada al colon.

217. Con respecto a las siguientes afirmaciones sobre la
pancreatitis aguda qu afirmacin es correcta:

1. Los diurticos tiazdicos se asocian a pancreatitis
aguda.
2. A la decoloracin alrededor del ombligo se le de-
nomina signo de Turner.
3. La presencia del signo de Cullen indica pancreati-
tis aguda edematosa.
4. La hiperamilasemia es un signo pronstico de la
pancreatitis aguda.
5. La hipertrigliceridemia suele ser consecuencia de
la inflamacin pancretica aguda.

218. La causa ms frecuente de aparicin de ascitis
pancretica es:

1. Rotura de conducto pancretico.
2. Hipertensin linftica.
3. Hipertensin venosa.
4. Fisuracin de un seudoquiste pancretico.
5. Ninguna de las anteriores.

219. Cul de los siguientes lbulos hepticos se ve
hipertrofiado de forma caracterstica en el Sd. de
Budd-Chiari? .

1. Lbulo de Spiegel.
2. Lbulo drcho.
3. Lbulo izdo.
4. Lbulo caudado.
5. Todos estn hipertrofiados por igual.

220. Hace un mes, una mujer de 21 aos comenz un
tratamiento diario con isoniacida debido a una cu-
tirreaccin tuberculnica positiva. Ella se siente
bien actualmente y su exploracin fsica es anodi-
na. Los anlisis rutinarios en suero indican: ALT:
150 U/L, bilirrubina total: 1 mg/dl y fosfatasa: 25
U/L. La decisin mdica ms adecuada es:

1. Dar otro frmaco antituberculoso.
2. Corticoides.
3. Biopsia heptica.
4. Administrar piridoxina intramuscular.
5. Continuar el tratamiento con isoniacida.

221. Cul de los siguientes signos cutneos es menos
tpico de cirrosis heptica?

1. Livedo reticularis.
2. Araas vasculares.
3. Distribucin feminoide del vello en el hombre.
4. Ictericia.
5. Eritema palmar.

222. El tratamiento con fenobarbital es ineficaz redu-
ciendo la hiperbilirrubinemia en:

1. Sndrome de Crigler Najjar tipo I.
2. Sndrome de Crigler Najjar tipo II.
3. Enfermedad de Gilbert.
4. Enfermedad de Caroli.
5. Dficit de alfa 1 antitripsina.

223. Un nio de 4 aos en quien se han observado sig-
nos de desarrollo sexual precoz presenta un alar-
mante estado general con anorexia, decaimiento y
prdida de peso. Se observa distensin abdominal
y en la ecografa se comprueba la presencia de un
tumor slido heptico. Cul de las siguientes de-
terminaciones tendra ms utilidad en este caso?

1. Hemograma completo y frmula leucocitaria.
2. Determinacin en orina de 17 cetosteroides.
3. Determinacin de los niveles sricos de antgeno
carcinoembrionario (CEA).
4. Determinacin de alfa fetoprotena srica.
5. TC craneal en bsqueda de tumor pineal.

224. NO suele presentarse en la colecistitis aguda no
complicada:

1. Signo de MURPHY.
2. Discreta elevacin de la bilirrubina.
3. Fiebre de 40C con escalofrios.
4. Antecedentes de clicos biliares.
5. Elevacin de la amilasa.

225. Ante un paciente que presenta un cuadro consis-
tente en ictericia, prdida importante de peso y
anorexia en las ltimas semanas, y que en la explo-
racin se palpa una vescula biliar distendida,
nuestra primera sospecha diagnstica debe ir
encaminada a descartar el grupo de los llamados
carcinomas periampulares. Cul de los siguientes
no se encuentra entre ellos?

1. Carcinoma del coldoco.
2. Carcinoma de la bifurcacin de los conductos
hepticos (Tumor de Klatskin).
3. Carcinoma de la cabeza del pncreas.
4. Carcinoma de la ampolla de Vater.
5. Adenocarcinoma duodenal.

226. Una mujer de 48 aos con molestias disppticas
atribuidas a colecistitis crnica se realiza
colecistectoma por laparoscopia. Cuatro meses
despus acude a urgencias con cuadro de fiebre
alta, escalofros, dolor en hipocondrio derecho e
ictericia con 9 mg/dl de bilirrubina total. Qu
explicacin es la ms probable en este caso?

1. El diagnstico de colecistitis crnica haba sido
errneo.
2. Exista colecistitis crnica, pero los sntomas de la
paciente no eran debidos a ello.
3. Al no explorarse las vas biliares previamente al
acto quirrgico mediante CPRE no se apreciaron
clculos existentes en el coldoco.
4. Se ha producido una dehiscencia de la sutura.
5. El cuadro es atribuible a una estenosis orgnica del
esfnter de Oddi.

227. Mientras su marido est en el Caribe, la mujer de
Curro acude a su mdico por sentir los mismos
Centro de Estudios Aula Neptuno - GRANADA Ver.: 07/2000
www.aulaneptuno.com
- 25 -
sntomas disppticos que se haban atribuido a li-
tiasis vesicular y que le haban hecho someterse a
una colecistectoma. Cul es el origen ms proba-
ble de los sntomas de esta paciente?

1. Depresin y somatizacin por abandono conyu-
gal.
2. Clonorquiasis transmitida a distancia.
3. Desorden extrabiliar presente antes de la ciruga.
4. Coledocolitiasis residual.
5. Colangiocarcinoma.

228. No es indicacin de endoscopia en pacientes con
lcera duodenal:

1. Detectar una posible lcera duodenal ante un pa-
ciente con sntomas tpicos en quien la lesin no se
ha podido demostrar por radiologa con contraste.
2. Persistencia de los sntomas tras una semana de
tratamiento mdico.
3. En caso de hemorragia gastrointestinal aguda.
4. Pacientes con deformidad radiolgica importante
que despierta dudas sobre la actividad de una lce-
ra.
5. Identificacin de lceras demasiado pequeas o
superficiales para ser detectadas en la radiologa.

229. La administracin de uno de los siguientes prepa-
rados farmacolgicos tiene como efecto secundario
una estimulacin de la secrecin cida gstrica
(efecto rebote), cul es?

1. Omeprazol.
2. Sucralfato.
3. Pirencepina.
4. Carbonato de calcio.
5. Acexamato de cinc.

230. En cul de estas enfermedades que predisponen
al cncer colorrectal hay menos riesgo relativo de
padecer el mismo?

1. Poliposis colnica juvenil.
2. Ureterosigmoidostoma.
3. Sndrome de Turcot.
4. Colitis ulcerosa.
5. Adenomas vellosos.

231. Qu es falso respecto al la Enfermedad de Gil-
bert?

1. Es la causa ms frecuente de ictericia metablica
constitucional.
2. Se debe a mltiples deficiencias del metabolismo
de la bilirrubina, siendo la ms importante la alte-
racin de la funcin excretora.
3. Se caracteriza por ictericia fluctuante que aumenta
con el ayuno y con las situaciones estresantes que
sufre el organismo.
4. Para diferenciarlo de las anemias hemolticas se
puede utilizar una prueba del ayuno.
5. Aunque los barbitricos disminuyen la bilirrubina
no estn indicados para el tratamiento porque es
un trastorno benigno.

232. Indica la falsa sobre las enfermedades vasculares
del intestino delgado y colon:

1. La prueba inicial que debe realizarse en todo pa-
ciente con sospecha fundada de colitis isqumica
segmentaria es la arteriografa mesentrica.
2. EL tratamiento de la colitis isqumica segmentaria
debe ser conservador con antibiticos y reposicin
hidroelectroltica.
3. La angiodisplasia de colon puede diagnosticarse
mediante colonoscopia o arteriografa que, a su
vez, pueden ayudar al tratamiento hemosttico.
4. El infarto intestinal agudo puede diagnosticarse
con ECO-doppler y arteriografa.
5. Debe sospecharse isquemia intestinal crnica si se
presentan dolores abdominales postprandiales de
localizacin periumbilical.

233. En relacin con el ejercicio en la diabetes melllitus,
slo una es cierta:

1. El ejercicio siempre est indicado en la diabetes
mellitus.
2. La aplicacin de insulina debe realizarse preferen-
temente en la pierna o el brazo.
3. La respuesta de la glucemia siempre es el descen-
so en el ejercicio.
4. La hipoglucemia se produce unos 30' despus de
iniciar el ejercicio.
5. Los pacientes con retinopata proliferativa no de-
ben hacer ejercicio.

234. De las siguientes afirmaciones sobre la insulinote-
rapia, una es falsa:

1. El diabtico tipo I debe controlarse habitualmente
con insulina rpida intravenosa.
2. La cantidad necesaria en un DMID suele oscilar
de 0,5-1 U/Kg de peso.
3. La dosis total de insulina en un DM tipo II puede
darse en una sola dosis.
4. En el tratamiento de la cetoacidosis se corrige ms
rpidamente la hiperglucemia que la cetosis.
5. En el debut de un diabtico se debe empezar con
insulina humana.

235. Una enfermera de 40 aos ha tenido episodios de
confusin, diaforesis y palpitaciones durante las 4
ltimas semanas. Ha tenido varias pesadillas y tres
episodios sincopales. En el hospital se comprueba
una hipoglucemia en ayunas, con una concentra-
cin plasmtica del pptido C tambin elevada. Su
mdico debe :

1. Medir la concentracin plasmtica de Ac anti-
insulina.
2. Sospechar el empleo de insulina exgena.
3. Medir la concentracin plasmtica y urinaria de
sulfonilureas.
4. Hacer un TAC abdominal de inicio.
5. Consultar con un cirujano para una intervencin
quirrgica de pncreas.

Centro de Estudios Aula Neptuno - GRANADA Ver.: 07/2000
www.aulaneptuno.com
- 26 -
236. Un enfermo diabtico con retinopata proliferativa
sufre brusca prdida de visin del ojo derecho sin
otros sntomas neurolgicos asociados. Tu primera
sospecha diagnstica ser:

1. ACV.
2. Glaucoma agudo.
3. Agravamiento de la retinopata diabtica.
4. Oclusin de la arteria central de la retina.
5. Hemorragia vtrea o desprendimiento de retina.

237. Se han descrito hiperlipemias asociadas al uso de
diferentes frmacos, cul de los citados no las
produce?

1. Tiacidas.
2. Betabloqueantes.
3. Esteroides.
4. Captopril.
5. Contraceptivos orales.

238. Indica qu afirmacin es falsa sobre la fisiologa
renal del potasio:

1. El 30-50% del potasio filtrado se reabsorbe por el
tbulo proximal.
2. El aumento de aporte de sodio al tbulo distal con-
lleva una disminucin de la excrecin de potasio.
3. En la porcin gruesa ascendente del asa de Henle,
el potasio se reabsorbe junto con el sodio por un
mecanismo acoplado a la reabsorcin del cloro.
4. El aumento del pH plasmtico facilita la secrecin
de potasio en la orina.
5. En la diuresis osmtica tambin se favorece la ex-
crecin de potasio.

239. Qu afirmacin es falsa en relacin con el trata-
miento de la porfiria cutnea tarda?

1. La prctica de flebotomas repetidas constituye el
tratamiento de eleccin, ya que el vaciamiento de
los depsitos corporales de hierro libera a la uro-
porfiringeno descarboxilasa heptica de los me-
canismos de inhibicin a que se encuentra someti-
da.
2. La desferrioxamina es un quelante del hierro de
fcil administracin gracias a su larga vida media,
que permite el tratamiento con dosis semanales del
frmaco.
3. Son recomendables las sangras trimestrales de
mantenimiento, aunque la enfermedad siga en
remisin.
4. La cloroquina forma complejos solubles con las
porfirinas acumuladas, favoreciendo as su excre-
cin.
5. Durante la fase bioqumicamente activa de la en-
fermedad, el paciente deber protegerse de la ex-
posicin al sol.

240. Mujer de 36 aos que , tras discusin con su mari-
do a consecuencia de disparidad de criterios sobre
la importancia del encuentro Rumana-Suecia de
ftbol, en comparacin con "Lo que necesitas es
amor", presenta crisis de ansiedad, con marcada
hiperventilacin y espasmo carpopedal. A la explo-
racin destacan marcas de araazos en la mejilla
derecha del marido. La gasometra de la seora
presenta un PO2 de 107 mm.Hg, PCO2 de 18
mm.Hg, HCO3 de 24 mm.Hg, y pH de 7,645. Po-
demos definir la situacin como:

1. Alcalosis reactiva a acidosis metablica.
2. Alcalosis metablica y respiratoria.
3. Alcalosis metablica compensada.
4. Alcalosis respiratoria aislada.
5. Acidosis respiratoria hipercompensada.

241. En uno de los siguientes sndromes neurolgicos es
ms probable encontrar una expresin exagerada
de la afectividad:

1. Talmico.
2. Frontal.
3. Occipital.
4. Cerebeloso.
5. Seudobulbar.

242. En la esquizofrenia, la hiperactividad noradrenr-
gica se ha relacionado con la aparicin de snto-
mas:

1. Catatnicos.
2. Tipo mutismo.
3. Paranoides.
4. Indiferencia emocional.
5. De desorganizacin.

243. Uno de estos individuos no tiene un trastorno de la
personalidad perteneciente al grupo excntrico:

1. Individuo que se queja de ser injustamente tratado
por todas las instancias superiores.
2. Individuos fros, aislados del resto de personas que
ocasionalmente tienen cerca.
3. Individuo que dice poseer poderes sobrenaturales.
4. Individuo que intenta exhibirse en cualquier reu-
nin en que se encuentre.
5. Individuos desconfiados y reivindicativos.

244. Sobre el manejo teraputico de los alcohlicos, un
enunciado no es correcto:

1. Las curas de desintoxicacin utilizan diferentes
psicofrmacos sedantes y tienenn una duracin de
unos 11 das.
2. Posteriormente a las curas de desintoxicacin, se
realizan las curas de deshabituacin, en las que
pueden usarse los frmacos interdictores, tipo di-
sulfiram.
3. El enfermo debe permanecer ignorante respecto a
los tratamientos conductuales, que debern con-
sensuarse con el familiar ms cercano.
4. En el tratamiento de mantenimiento, es
fundamental el papel de las terapias grupales.
5. Un sndrome de abstinencia muy grave debe ser
tratado en medio hospitalario.

245. Son frecuentes en la drogadiccin por alucinge-
Centro de Estudios Aula Neptuno - GRANADA Ver.: 07/2000
www.aulaneptuno.com
- 27 -
nos, las siguientes alteraciones, excepto una:

1. Importantes oscilaciones del humor y del estado
de nimo.
2. Perturbacin y distorsin del espacio-tiempo y del
esquema corporal.
3. Ideacin delirante de tipo autorreferencial y msti-
co.
4. Reaparicin de la sintomatologa aguda de modo
espontneo (fenmeno de flash-back).
5. Miosis, bradicardia e hiporreflexia.

246. Sobre las pautas teraputicas en el tratamiento con
antidepresivos, qu proposicin de las siguientes
es incorrecta?

1. Una vez se abandona un tricclico su efecto dura
an unas tres semanas, tras las cuales hay un ries-
go de recada.
2. La interrupcin brusca de un tricclico no produce,
en general, abstinencia o rebote, pero a veces se
han descrito dolor abdominal, diarreas, insomnio y
ansiedad, por lo que recomendaremos una pauta
descendente.
3. Cuando se sustituye un tricclico por un IMAO
deben transcurrir al menos dos semanas de la total
interrupcin del primero, y ms de 5 semanas si se
trata de fluoxetina o paroxetina.
4. Si hay mejora en dos semanas, el tratamiento
puede empezar a retirarse al mes de haberlo ini-
ciado.
5. Cuando se cambia un tricclico clsico por fluoxe-
tina no es necesario ningn periodo de lavado en-
tre uno y otro.

247. Dentro de los datos bsicos que se deben obtener
inicialmente de forma sistemtica ante un paciente
con un sndrome ansioso, no se encuentra:

1. Perfil glucmico.
2. Exploracin neurolgica.
3. Palpacin de tiroides.
4. Auscultacin cardiaca.
5. Entrevista sobre datos familiares (feocromocito-
ma, MEN, ansiedad y depresin...).

248. La actual consideracin de las crisis de angustia es
que se originan por:

1. Conflictos psicolgicos inconscientes.
2. Mecanismos endgenos independientes de los es-
tmulos ambientales.
3. Conflictos infantiles que se reagudizan en la vida
adulta.
4. Circunstancias ambientales adversas.
5. Escasa tolerancia a las frustraciones.

249. Cul es el tratamiento farmacolgico ms ade-
cuado para el trastorno por dficit de atencin?

1. Clomipramina.
2. Haloperidol.
3. Diacepam.
4. Metilfenidato.
5. Amitriptilina.

250. La opinin ms actual y generalizada sobre el
autismo infantil es que se trata de:

1. Una alteracin del contacto socioafectivo exclusi-
vamente.
2. Un trastorno de la atencin.
3. Un tipo especial de neurosis.
4. Un trastorno esquizoafectivo.
5. Un trastorno generalizado del desarrollo.

251. Un paciente de 65 aos, sin antecedentes de inters,
excepto cirrosis heptica por virus C, consulta por
disnea de esfuerzo. La gasometra muestra: pH
7.4; pCO2 35 mmHg; PO2 58 mmHg y PO2 (Aa)
48, no mejorando tras oxigenoterapia, La radio-
grafa de trax es normal. Cul es, entre los si-
guientes, el trastorno subyacente responsable?:

1. Alteraciones de la pared torcica.
2. Shunt intrapulmonar.
3. Enfermedad intersticial pulmonar.
4. Enfermedad vascular pulmonar.
5. Enfermedad obstructiva de la va area.

252. Cual dc las siguientes aseveraciones relativas a la
insuficiencia respiratoria es correcta?:

1. Conceptualinente se considera que existe cuando
la PO2 es inferior a 70 mmHg.
2. Su mecanismo fisiopatolgico ms frecuente es
una discordancia entre la ventilacin y la perfli-
sin.
3. Se produce con ms frecuencia por una dificultad
de la capacidad de diflisin alveolo-capilar del O2.
4. Conceptualmente se considera que existe cuando
la PCO2 es superior a 42 mmHg.
5. Produce una disminucin de la diferencia alveolo
arterial de O2 (PAO2-PaO2).

253. Una mujer de 86 aos hospitalizada por un acci-
dente cerebrovascular, es dada de alta envindole a
un centro de media estancia para rehabilitacin f-
sica. A la exploracin destaca una prdida de fuer-
za en hemicuerpo izquierdo grado 4/5 y mnimo
dficit sensitivo. En una exploracin rutinaria rea-
lizada hace 10 meses se detect fibrilacin auricu-
lar asintomtica. Se realiz ecocardiograma que
mostr una aurcula izquierda de 6,5 cm. de di-
metro. No se inici ningn tratamiento. Tomaba
hicroclorotiazida y captopril para hipertensin ar-
terial. Entre los siguientes, cul es el paso ms in-
dicado en el manejo de esta paciente?:

1. Aspirina 100 mg al da.
2. Dipiridamol 150 mg al da.
3. Acenocumarol.
4. Cardioversin elctrica.
5. Ecocardiograma transesofgico para demostrar un
trombo auricular.

254. Seale la asociacin FALSA en la exploracin
clnica de pacientes con valvulopatas:
Centro de Estudios Aula Neptuno - GRANADA Ver.: 07/2000
www.aulaneptuno.com
- 28 -

1. Pulso Parvus-Estenosis artica.
2. Pulso Tardus-Insuficiencia artica.
3. Chasquido de apertura-Estenosis mitral.
4. Soplo Mesosistlico-Estenosis artica.
5. Soplo Pansistlico-Insuficiencia mitral.

255. Por qu es importante estandarizar las tasas
de mortalidad por edad?:

1. Para ofrecer una informacin ms detallada.
2. Para conseguir tasas especficas por grupos de
edad.
3. Para obtener una cifra real de mortalidad.
4. Para poder comparar tasas de diferentes lugares
eliminando la confusin que general el factor
edad.
5. Para obtener una medida cruda de la mortalidad.

256. La biopsia intestinal, realizada por un cuadro de
malabsorcin en un adulto joven, muestra abun-
dantes macrfagos PAS positivos en la lmina
propia Cul es el diagnstico ms probable?:

1. Enfermedad celiaca del adulto.
2. Enfermedad de Whipple.
3. Abetalipoproteinemia.
4. Agammaglobulinemia.
5. Infeccin por micobacteria atipica.

257. Un joven de 27 aos llega a Urgencias con protru-
sin de lengua y espasmo de torsin de cuello. Un
familiar que le acompaa slo sabe decir que re-
cientemente tuvo un breve ingreso psiquitrico y le
han puesto un tratamiento inyectable. En esta si-
tuacin, el diagnstico ms probable es:

1. Tetania.
2. Distonia aguda inducida por neurolpticos.
3. Trastorno por ansiedad aguda.
4. Corea de Huntington.
5. Trastorno por simulacin.

258. Hombre de 42 aos que, un ao antes, present
lesiones eritematosas, descamativas y pruriginosas
en superficie extensora de ambos codos. Seis meses
despus not hinchazn y dolor en interfalngica
del primer dedo de mano derecha y en interfaln-
gicas (IF) de 2, 3 y 5 dedos de pie derecho. Au-
sencia de antecedentes de pleuritis, diarrea, lum-
balgia, citica, iritis y uretritis. El examen fsico
muestra las lesiones cutneas referidas en codos y
en el lado derecho del cuero cabelludo. Junto a los
fenmenos inflamatorios articulares descritos, se
comprueba deformidad en "salchicha" (dactilitis)
en IF de 3 y 4 dedos de pie izquierdo. Las uas
presentan lesiones punteadas, onicolisis y surcos
horizontales palpables. Datos de laboratorio:
ANA, factor reumatoide y serologa de les negati-
vos; hiperuricemia moderada y nivel elevado de
IgA; PCR++, V. Sed. 30 mm en 1 hora. RX de
manos: pequeas erosiones perarticulares en IF
distales de 2 y 3 dedos de mano derecha y suge-
rencia de osteolisis de falanges distales. Con ms
probabilidad el paciente tendr:

1. Artritis reumatoide.
2. Artritis psorisica.
3. Sndrome de Reiter.
4. Artropatia amiloide.
5. Artritis asociada a enfermedad inflamatoria intes-
tinal.

259. Cul es la complicacin ms frecuente en los
aneurismas arteriosclerosos de aorta abdominal
mayores de 6 cm de dimetro?:

1. La oclusin de arterias mesentricas.
2. La embolizacin distal.
3. La compresin ureteral con hdronefrosis secunda-
ria.
4. La fistula aortocava.
5. La ruptura.

260 Cul de las siguientes pruebas diagnsticas NO es
imprescindible para el estudio bsico de la pareja
estril?:

1. Cariotipo.
2. Histerosalpingografa.
3. Analtica hormonal.
4. Biopsia de endometrio.
5. Seminograma.


CORRECCION

n de Preg. CONTESTADAS: ______ !
n de Preg. ACERTADAS: ______ "
n de Preg. FALLADAS: _____ #=!- "


TOTAL Puntuacion MIR: _________ "-(#/3)
Nombre: ___________________________
Apellidos: ___________________________
Fecha: ____ / _______________ / 2001
1 2 3 4 5 6 7 8 9 10 11 12 13 14 15 16 17 18 19 20
5 2 3 5 1 3 2 4 5 3 4 3 5 5 4 3 2 4 1 4
21 22 23 24 25 26 27 28 29 30 31 32 33 34 35 36 37 38 39 40
4 3 5 4 4 5 5 2 5 5 2 5 4 5 5 1 4 1 1 1
41 42 43 44 45 46 47 48 49 50 51 52 53 54 55 56 57 58 59 60
4 4 2 3 1 4 4 1 3 1 3 2 4 4 2 1 4 4 2 5
61 62 63 64 65 66 67 68 69 70 71 72 73 74 75 76 77 78 79 80
1 5 3 3 5 2 1 1 4 4 5 4 2 4 5 4 1 3 1 4
81 82 83 84 85 86 87 88 89 90 91 92 93 94 95 96 97 98 99 100
2 2 4 3 3 3 2 3 2 3 1 2 3 4 5 5 3 1 4 1
101 102 103 104 105 106 107 108 109 110 111 112 113 114 115 116 117 118 119 120
1 4 1 4 1 1 3 2 1 4 3 5 5 2 2 3 5 2 2 4
121 122 123 124 125 126 127 128 129 130 131 132 133 134 135 136 137 138 139 140
5 5 1 1 1 5 3 3 4 4 1 2 3 2 4 1 1 4 1 4
141 142 143 144 145 146 147 148 149 150 151 152 153 154 155 156 157 158 159 160
5 4 5 1 1 4 5 3 1 2 5 4 1 5 4 5 5 1 2 4
161 162 163 164 165 166 167 168 169 170 171 172 173 174 175 176 177 178 179 180
5 3 3 5 1 3 3 4 3 1 4 4 3 2 5 5 2 3 2 1
181 182 183 184 185 186 187 188 189 190 191 192 193 194 195 196 197 198 199 200
5 3 3 2 2 5 2 5 5 1 4 3 3 2 3 5 4 5 4 5
201 202 203 204 205 206 207 208 209 210 211 212 213 214 215 216 217 218 219 220
1 2 4 2 3 4 2 4 2 5 5 1 2 4 3 3 1 4 4 5
221 222 223 224 225 226 227 228 229 230 231 232 233 234 235 236 237 238 239 240
1 1 4 3 2 3 3 2 4 1 2 1 5 1 3 5 4 2 2 4
241 242 243 244 245 246 247 248 249 250 251 252 253 254 255 256 257 258 259 260
5 3 4 3 5 4 1 2 4 5 2 2 3 2 4 2 2 2 5 1
Hoja de control
del % de aciertos por materias
en Exmenes MIR AULA NEPTUNO
MATERIA PREGUNTAS N: N PREGUNTAS N ACIERTOS factor% % de Aciertos
TRAUMATOLOGI A 1 a 8 8 12,50
M.LEGAL Y TOX. 9 a 10 2 50,00
GI NECOLOGI A 11 a 14 4 25,00
ENF. INFECCIOSAS 15 a 40 26 3,85
OFTALMOLOGI A 41 a 45 5 20,00
PEDIATRIA 46 a 49 4 25,00
NEUROLOGI A 50 a 69 20 5,00
NEUMOLOGIA 70 a 89 20 5,00
REUMATOLOGI A 90 a 103 14 7,14
CIRUGIA 104 a 113 10 10,00
ENDOCRI NOLOGI A 114 a 125 12 8,33
EPIDEMI OLOGIA 126 a 134 9 11,11
FI SI OLOGI A 135 a 139 5 20,00
DERMATOLOGI A 140 a 144 5 20,00
NEFROLOGIA 145 a 162 18 5,56
O.R.L. 163 a 167 5 20,00
OBSTETRI CIA 168 a 171 4 25,00
BI OESTADI STI CA 172 a 180 9 11,11
CARDIOLOGIA 181 a 190 10 10,00
HEMATOLOGIA 191 a 210 20 5,00
AP. DI GESTI VO 211 a 232 22 4,55
METABOLISMO 233 a 240 8 12,50
PSIQUIATRIA 241 a 250 10 10,00
Para calcular el % de aciertos en cada materia de este examen MIR,
corrige tu examen teniendo en cuenta la siguiente tabla, con la que podrs
identificar las preguntas que corresponden a cada tema.
Anota el n de respuestas que has acertado en la columna
n Aciertos y multiplica por el factor% anotando el resultado de la
operacion en la columna % de aciertos (puede haber un mnimo error por redondeo)
As podrs comprobar en qu materias aciertas ms preguntas. Anota
al final de la hoja las 5 en las que hayas obtenido porcentajes ms bajos y
tenlas en cuenta para tus prximos repasos.

También podría gustarte